Rad Onc ACR Questions Lung/Thorax/Thymoma

अब Quizwiz के साथ अपने होमवर्क और परीक्षाओं को एस करें!

Regarding The National Lung Cancer Screening Trial, the rate of: (A) lung cancer-specific mortality was decreased 20% in the low-dose CT group. (B) death from any cause was the same in both groups. (C) adherence to screening was 50%. (D) false positives were lower in the low-dose CT group.

A National Lung Screening Trial (NLST), 2011 "Reduced Lung-Cancer Mortality with Low-Dose Computed Tomographic Screening" (NEJM. 2011 Aug 4;365(5):395-409.) Age 55 and 74 year of age, *30 pack year hx of smoking and if quit then w/in 15 years*; at high risk for lung cancer, randomly assigned to undergo 3 annual screenings with either low-dose CT or single-view PA chest radiography. Outcome:Rate of positive screening tests: 24.2% (low-dose CT) vs. 6.9% (radiography); False positive: 96.4% vs. 94.5%; *relative reduction in mortality from lung cancer with low-dose CT screening. 6.7% reduction in all cause mortality.*

According to the latest AJCC staging manual for lung cancer, which of the following would be classified as M1a? (A) Malignant pleural effusion (B) Separate tumor nodule(s) in the same lobe (C) Contralateral supraclavicular lymph node involvement (D) Separate tumor nodule(s) in a different ipsilateral lobe

A pM1 Distant metastasis, microscopically confirmed pM1a -Separate tumor nodule(s) in a contralateral lobe; -tumor with pleural or pericardial nodules o *malignant pleural or pericardial effusion *microscopically confirmed. Most pleural (pericardial) effusions with lung cancer are a result of the tumor. In a few patients, however, multiple microscopic examinations of pleural (pericardial) fluid are negative for tumor, and the fluid is nonbloody and not an exudate. If these elements and clinical judgment dictate that the effusion is not related to the tumor, the effusion should be excluded as a staging descriptor. pM1b Single extrathoracic metastasis in a single organ (including involvement of a single nonregional node), microscopically confirmed pM1c Multiple extrathoracic metastases in a single organ or in multiple organs, microscopically confirmed

What would be the cure rate for patients with stage III NSCLC treated with either surgery or radiation therapy alone? (A) <10% (B) 20% (C) 30% (D) 50%

A About 150,000 new cases of NSCLC are diagnosed every year in the United States. At least 40% will present with Stage III NSCLC. Surgical resection or radiotherapy alone will result in cure in less than 10% of patients (Mountain CF. Revisions in the international staging system for staging lung cancer. Chest 111:1710-1717, 1997).

In patients with early-stage NSCLC planned for surgical resection: (A) a minimum of 3 N2 stations should be sampled. (B) LRC rates with sub-lobar or lobar resection are similar. (C) complete mediastinal nodal dissection improves survival over sampling. (D) nodal stations 4, 5,6,7,8 and 9 should be sampled for right-sided tumors.

A In a randomized phase-III trial by LCSG, *sub-lobar resection resulted in a 75% increase in relative recurrence* rates versus lobar resection (Ginsberg et al). In a randomized trial (ACOSOG Z0030), *no added benefit of completion mediastinal nodal dissection if thorough nodal sampling was negative*. For *right sided tumors 2R,4R, 7 and 10R were sampled*, while for l*eft-sided lesions 5,6,7 and 10L were sampled*. To improve predictive probability of sampling, NCCN also suggests *stations 8 and 9 sampling with a minimum of 3 N2 stations to be sampled*. Knowledge of adequate surgical management can guide adjuvant therapy.

Which of the following is TRUE regarding the INT 0139, a trial comparing neoadjuvant chemoradiation and surgical resection to definitive chemoradiation for locally advanced NSCLC? (A) The majority of the treatment-related deaths occurred in patients undergoing pneumonectomy. (B) The majority of the treatment-related deaths in both groups occurred during chemoradiation. (C) Both the lobectomy and pneumonectomy after the chemoradiation provided identical overall survival. (D) Chemotherapy for both groups consisted of carboplatin and taxol.

A In this pivotal trial, only patients with stage IIIA (N2) NSCLC were eligible. Progression-free survival was superior for neoadjuvant chemoradiotherapy (median 12.8 vs. 10.5 mo, p=0.017), though overall survival was not. In an unplanned analysis, overall survival was improved for neoadjuvant chemoradiotherapy for patients who underwent lobectomy but not pneumonectomy. *High rates of treatment related deaths occurred in patients undergoing pneumonectomy*.

Regarding the randomized trial investigating the role of thoracic RT for extensive-stage SCLC (Slotman), the addition of thoracic RT: (A) improved the 2-year OS. (B) did not improve 6 month PFS. (C) consisted of 45 Gy in 30 Fx over 3 weeks. (D) significantly increased Grade 3 or higher toxicity.

A In this study patients with extensive stage small cell lung cancer that had a response to chemotherapy were randomized to receive thoracic radiation therapy (30 Gy in 10 fractions) or no thoracic radiation therapy. All patients received prophylactic cranial irradiation (PCI). *The primary endpoint was 1 year overall survival which was similar between the two groups (33% vs 28%, p=0.066),* however a post-hoc secondary analysis revealed significantly greater *2-year overall survival in the thoracic radiation therapy group (13% vs 3%, p=0.004).* *Six-month progression-free survival was significantly improved in the thoracic radiation group (24% vs 7%, p=0.001)*. Grade 3 or higher *toxicity was similar* between the two groups (26 patients vs 18 patients, p=0.28)

In RTOG 0617 for patients with unresectable NSCLC who were treated with definitive chemoRT, which factor was significantly associated with OS on multivariate analysis? (A) Heart V30 (B) RT protocol compliance (C) RT technique (3DCRT vs IMRT) (D) Systemic chemotherapy dose compliance

A On multivariate analyses, *factors predicting overall survival* in RTOG 0617 were *radiation dose (60 Gy), maximum esophagitis grade, planning target volume, and heart V5 and V30*. *Radiation therapy and chemotherapy compliance, and radiation therapy technique* were *not significant for overall survival* in these analyses, however *IMRT did reduce pneumonitis rates*. Pts either 60 Gy (standard dose), 74 Gy (high dose), 60 Gy plus cetuximab, or 74 Gy plus cetuximab. Median overall survival was 28.7 months (95% CI 24.1-36.9) for patients who received standard-dose radiotherapy and 20.3 months (17.7-25.0) for those who received high-dose radiotherapy (hazard ratio [HR] 1.38, 95% CI 1.09-1.76; p=0.004). 74 Gy radiation given in 2 Gy fractions with concurrent chemotherapy was not better than 60 Gy plus concurrent chemotherapy for patients with stage III non-small-cell lung cancer, and might be potentially harmful. *Addition of cetuximab to concurrent chemoradiation* and consolidation treatment provided *no benefit in overall survival* for these patients.

All of the following statements regarding extensive stage SCLC are *CORRECT, EXCEPT*: (A) median survival time is around 20 months. (B) a complete response rate is estimated at 10-20%. (C) overall response rate to chemotherapy is 40-70%. (D) recurrence is the rule, even following an excellent response to initial chemotherapy.

A Overall response rate to multi-agent chemotherapy is 40-70% and complete response rate is estimated at 10-20%. Recurrence of disease is the rule, even following an excellent response to initial chemotherapy. Median survival time is around 9 to 10 months. The most common metastatic sites at diagnosis are: bone (19% to 38%), liver (17% to 34%), adrenal glands (10% to 17%), brain (0% to 14%).

What is the appropriate management for a patient with a history of metastatic NSCLC who presents with a 2 week history of bilateral lower extremity weakness and incontinence of bowel with imaging demonstrating spinal cord compression at T8? (A) RT alone to 30 Gy in 2 weeks (B) Transversectomy and postoperative RT (C) Second line chemotherapy (D) SBRT to 20 Gy in 1 fraction

A Palliative radiation for pain management is more appropriate in this case since the *duration of the symptoms does not justify using a surgical decompression approach as per the Patchell trial*. Patients who presents with symptoms *within 48 hours* may be *helped with surgical decompression* and postoperative radiotherapy. SBRT will not be appropriate for tumor mass compressing directly on the spinal cord. Chemotherapy will not address the cord compression or the pain.

A recent pooled analysis of two Phase-III studies (STARS and ROSEL) examining SABR/SBRT versus surgery for early stage NSCLC demonstrated: A) a statistically superior OS in favor of the radiation arm. B) a statistically reduced distant metastases in favor of the radiation arm. C)a statistically superior LRC in favor of the surgery arm. D) similar rates of grade 3-4 toxicity events in the two arms.

A SABR versus lobectomy for operable stage I non-small-cell lung cancer: a pooled analysis of two randomized trials (Chang JY et al. Lancet Oncol. 2015 Jun;16(6):630-7) - 58 patients, Estimated *overall survival at 3 years* was *95% in the SABR group* compared with *79% in the surgery group (p=0·037)*. *Recurrence-free survival at 3 years* was *86% in the SABR group* and *80% in the surgery group (p=0·54)*. *Grade 3-4 toxicity* rates were *44% in the surgery arm* versus *10% in the SABR arm.* - Critiques: small #s, 4% mortality from surgery (only 1 patient), why OS difference if no RFS difference?

What is the QUANTEC-defined cardiac dose for conventional fractionated 3D conformal radiation that results in < 1% risk of long term cardiac mortality? (A) V25 < 10% (B) V30 < 55% (C) V40 < 15% (D) Mean < 40 Gy

A The correct answer is V25 < 10% will result in a less than 1% risk of long term cardiac mortality. A Mean dose to the heart (pericardium) of < 26 Gy is associated with a <15% of pericarditis as is a V30 of <46% is also associated with a <15% risk of pericarditis.

About what percent of limited stage SCLC by conventional staging may be upstaged by FDG-PET? (A) 10 (B) 20 (C) 30 (D) 40

A. *FDG-PET correctly upstaged 8.3%* of limited staged SCLC patients to extensive- stage disease (initially diagnosed with limited disease small-cell lung cancer based on conventional staging). PET correctly identified tumor in each small-cell lung cancer mass (primary or nodal) that was suspected on computed tomography (CT) imaging, thus giving a *lesion-based sensitivity relative to CT of 100%*. FDG- PET has high sensitivity for small-cell lung cancer and appears to be of value for initial staging and treatment planning of patients with presumed limited-stage disease. FDG-PET scan detected additional lesions in patients diagnosed as having limited-stage small- cell lung cancer by conventional staging procedures. The therapeutic strategies were changed in 8% of patients based on the results of FDG-PET. *FDG-PET scan is recommended as an initial staging tool for patients with this disease.*

Immunohistochemical staining is important to distinguish mesothelioma from which of the following types of lung cancer? (A) Adenocarcinoma (B) Small cell carcinoma (C) Squamous cell carcinoma (D) Large cell neuroendocrine carcinoma

A. Diagnosis of mesothelioma can be difficult. Immunohistochemical staining is important to distinguish mesothelioma from adenocarcinomas of lung origin or metastatic from other sites. *Calretinin is commonly positive in mesothelioma*, with a reported sensitivity of 95% and specificity of 87%. *Thrombomodulin has the best specificity* at 92% but is less sensitive at 68%.

According to the randomized study (Turrisi, 1999) for limited stage SCLC, in comparison to once-daily radiation, twice-daily radiation had: (A) the 5-year OS of 26%. (B) a similar grade 3 esophagitis. (C) improved DFS, but not OS. (D) improved local control, but not OS

A. Intergroup Trial 0096, Turrisi (1999). 417 patients with LS-SCLC randomized to *concurrent cisplatin/etoposide* with either *45 Gy/1.8 Gy daily*, or *45 Gy/1.5 Gy BID.* a. RT fields: included bilateral mediastinum, ipsilateral hilum. Inferior border 5 cm below carina or including hilum. Elective ipsilateral SCV forbidden. b. Outcomes: i. Median OS daily arm: 19 months vs. BID arm: 23 months (SS). ii. 2-year OS 41% vs 47%. iii. 5-yr OS 16% vs 26% (SS). iv. Local failure daily arm 52% vs BID 36% (p=0.06). c. Toxicity:Grade 3 esophagitisin daily arm: *11% vs. 27%* BID, no difference in Grade 4 esophagitis. d. Conclusion: Survival exceeds that of any previous randomized trial. Small difference between QD and BID at 2 years but 10% improvement at 5 years. Comment: Not a comparison of once a day vs twice a day since the *biologically effective doses weren't equivalent*. 45 Gy BID is closer to *about 60 Gy QD,* which is the current "standard" dose (60-66 Gy) for once a day RT. However, at the time of the trial, 40-50 Gy QD was the standard dose *CONVERT *Tried to show this comparing 45Gy in 30fx (1.5 Gy BID) vs 66Gy in 33fx (2Gy daily) but 2yr OS 56% vs 51%, NS. This was *not an inferiority study* there for authors concluded *BID RT should remain standard of care*

Which is the correct stage for a NSCLC patient with a 4 cm tumor and a separate 2 cm tumor in the same lobe, an ipsilateral paratracheal lymph nodes, and no distant metastases? (A) T2bN1M0 stage IIB (B) T3N2M0 stage IIIA (C) T4N2M0 stage IIIB (D) T2bN2M0 stage IIIA

B

Regarding the role of adjuvant RT following complete surgical resection (negative margins) in a patient with NSCLC with evidence mediastinal lymph node involvement? (A) There is no role for RT following adjuvant chemotherapy. (B) RT should be delivered sequentially to decrease the morbidity. (C) The total dose should be 60 Gy since the tumor bed has been devascularized. (D) Chemotherapy should be administered concurrently to get the best local control and OS.

B *All the data demonstrating a benefit was seen in the sequentially treated patients* suggesting high complication rates related to concurrent approach (ECOG). Doses > 54 Gy have been shown to increase complications. 5-year DID rate: 8% control vs. 31% PORT (p=0.0001; RR 3.47) *<2 Gy-->16% vs 2 Gy 18% vs >2 Gyn 26% (Dautzenberg and Machtay).* *Adjuvant RT/Post-Operative RT (PORT):* - NOT recommended for N0-N1 disease b/c a/w increased mortality, at least when using older RT techniques - N2+ dz it improves survival as an adjunct to post op chemo (ANITA 2008), N2/N3 disease or positive margins (+/- ENE/ECE) - For T1a-T3N1, T3N0, N2: R1 resection: sequential chemoRT. R2 resection: concurrent chemoRT - RT usually follows chemo in post-op setting unless R2 resection. - If treating for positive margin, favor RT prior to chemo unless using concurrently - The role of ECE has not been well investigated. - PORT with concurrent chemo may be used in medically fit pts (Feigenberg et al, Bradley et al) - Recommended for positive surgical margins - If N2 margins are negative, adjuvant chemo for 2-4 C → mediastinal RT - Involved LN region +/- ipsilateral hilum +/- subcarinal LN region to 50.4 Gy depending on the extent of node dissection, number, bulk, and location of mediastinal disease and primary tumor -10-16Gy boost if extranodal extension - If positive margin (R1), favor initial post-op RT →adjuvant chemo or post op concurrent chemo-RT (e.g., carbo/taxol with RT). Limit field to area of +margin if N0-N1 disease (i.e. no elective mediastinal nodal coverage). -If gross residual disease (R2)→ favor concurrent chemo-RT

What is the preferred treatment approach for a patient diagnosed epithelioid mesothelioma with involvement of a mediastinal lymph node without evidence of metastatic disease? (A) Observation (B) Combination chemotherapy (C) Extrapleural pneumonectomy followed by adjuvant chemotherapy and RT (D) Decortication and RT

B *Patients with N+ nodal disease with malignant mesothelioma should not be offered surgical options*. Palliative chemotherapy using *combined chemotherapy with cisplatin and pemetrexed* should be considered the *standard of care*.

For a patient with histologic findings of neoplastic thymic epithelial cells with spindle shape, with great vessel invasion, what is the WHO Histologic Classification and Modified Masaoka stage? a. Type A, Stage IIB b. Type A, Stage IIIB c. Type B, Stage IIIA d. Type B, Stage IVA

B *Type A* has neoplastic thymic epithelial cells with *spindle/oval shape*. Type AB has features of type A admixed with *foci rich in lymphocytes*. Type B1 resembles *normal functional thymus*, Type B2 has scattered plump cells with vesicular nuclei among a *heavy population of lymphocytes* Type B3 is predominantly composed of round or polygonal shape with minimal atypia Type C is a *thymic carcinoma* with atypia with cytoarchitectural features no longer specific to the thymus. Modified Masaoka stage Stage I-Macroscopically *completely encapsulated Microscopically no capsular invasion Stage II Complete transmural *(transcapsular) invasion* -IIA. Microscopic *transcapsular invasion only* -IIB. Macroscopic invasion into extracapsular soft tissue, or *tumor grossly adherent* to mediastinal pleura or pericardium *w/o invasion* through these structures Stage III-*Macroscopic invasion* into neighboring organs -IIIA. Invasion *spares the great vessels* -IIIB. Invasion *includes the great vessels* Stage IVa- Pleural or pericardial implant or dissemination Stage IVb- Lymphogenous or hematogenous metastasis

Regarding role of extrapleural pneumonectomy (EPP) in patients with mesothelioma: a. localized sarcomatoid mesothelioma should best be managed with EPP, chemotherapy and adjuvant RT b. pleurectomy/decortication followed by pleural based IMRT to 45 Gy in 25 fractions has similar outcomes than EPP followed by adjuvant RT c. post-operative radiation dose after EPP with negative margins is 60 Gy d. positive cytology for malignant cells in the pleural fluid is a contraindication for EPP

B A. *Sarcomatoid mesothelioma has very poor prognosis* and per NCCN is considered a *relative contraindication for surgery*. Chemotherapy is the appropriate option for these patients, and palliative radiation as indicated. B. Per single institutional experience from MSKCC and a multicentre phase II study that used pleural based IMRT following pleurectomy/decortication for localized mesothelioma is an appropriate and safe option for therapy. Results appear promising and may be safer and more effective than historical data with EPP followed by adjuvant radiation. C. Typical adjuvant RT dose after *EPP with negative margins is 50-54 Gy*, while with *positive margins is 54-60 Gy*. D. Patients with localized mesothelioma often presents with pleural effusion. *Patients with malignant cells in the pleural fluid is not considered metastatic* and should be appropriately managed.

According to the latest AJCC staging manual for lung cancer, which of the following would be classified as a T3 tumor? (A) A Tumor >4 cm but ≤5 cm in greatest dimension (B) Separate tumor nodule(s) in the same lobe (C) Involvement of the main bronchus, ≥ 2 cm distal to the carina (D) Visceral pleura invasion

B AJCC 8 T1-Tumor ≤3 cm in greatest dimension, surrounded by lung or visceral pleura, without bronchoscopic evidence of invasion more proximal than the lobar bronchus (i.e., not in the main bronchus) T2-Tumor >3 cm but ≤5 cm or having any of the following features: • Involves the main bronchus regardless of distance to the carina, but without involvement of the carina • Invades visceral pleura (PL1 or PL2) • Associated with atelectasis or obstructive pneumonitis that extends to the hilar region, involving part or all of the lung T2 tumors with these features are classified as T2a if ≤4 cm or if the size cannot be determined and T2b if >4 cm but ≤5 cm. T3-Tumor >5 cm but ≤7 cm in greatest dimension or directly invading any of the following: •Parietal pleura (PL3), •Chest wall (including superior sulcus tumors), •Phrenic nerve, •Parietal pericardium; or •*Separate tumor nodule(s) in the same lobe as the primary" T4-Tumor >7 cm or tumor of any size invading one or more of the following: diaphragm, mediastinum, heart, great vessels, trachea, recurrent laryngeal nerve, esophagus, vertebral body, or carina; separate tumor nodule(s) in an ipsilateral lobe different from that of the primary

Which action decreases the risk of radiation pneumonitis? (A) Hypofractionation (B) Continued smoking (C) Concurrent chemotherapy (D) Concurrent immunotherapy

B Continued smoking after treatment is associated with a decreased risk for radiation pneumonitis, possibly because it *suppresses radiation-induced inflammation*. *Chemotherapy, immunotherapy and hypofractionation* could *increase the likelihood of pneumonitis, not decrease it.*

Regarding limited stage SCLC, chemoRT demonstrated: (A) a 5.4% improvement in OS compared with RT alone. (B) a 5.4% improvement in OS compared with cisplatin-based chemotherapy alone. (C) a 5.4% improvement in LRC, but no improvement in OS compared with RT alone. (D) a 5.4% improvement in LRC, but no improvement in OS compared with cisplatin-based chemotherapy alone.

B Due to high locoregional failure rates after chemotherapy alone, thoracic radiation in combination with chemotherapy was investigated for patients with limited stage disease. Several randomized studies compared chemotherapy alone to chemotherapy and radiation.

Regarding the use of adjuvant chemotherapy following surgical resection for lung cancer: (A) there is an improvement in OS of 10%, based on the LACE meta-analysis. (B) the combination of cisplatin and vinorelbine demonstrated the largest benefit in OS. (C) in the absence of nodal involvement, CALGB 9633 demonstrated an improvement in OS. (D) the combination of carboplatin and paclitaxel has the largest relative improvement in OS.

B In the most recent update of CALGB 9633, which was the only phase III trial to test the combination of carboplatin and paclitaxel, there was *no difference in outcomes for tumors > 3 cm without nodal involvement*. There is a suggestion of an improvement in patients with *tumors > 4 cm but this was based on an unplanned subset analysis*. LACE meta-analysis of 4,584 found that postoperative cisplatin-based chemotherapy *increased absolute 5 yr OS by 5.4%* · No difference is different chemotherapy regiments · *A subgroup analysis found (vinorelbone/cisplatin) improved survival* · Benefit greatest in Stage II and III disease with good KPS · Postop chemotherapy benefited elderly pts up to 80 years of age!

According to Kalapurakal, et al in IJROBP 2013, what is the PRINCIPAL organ spared by IMRT instead of AP-PA fields for pediatric whole lung irradiation? (A) Esophagus (B) Heart (C) Liver (D) Thyroid

B Kalapurakal, et al, showed that the dose to the heart could be significantly reduced while improving PTV whole lung coverage. However, dose to the liver and thyroid were not significantly different. Esophagus was not assessed in the manuscript but is relatively small in cross sectional area and effectively included in whole lung PTV.

In the Phase-I RTOG 0813 study that examined SBRT for centrally located NSCLC treated in 5 fractions: a. the MTD was 10 Gy/ fx. b. the 2-year LC at the two highest dose-cohorts was 85-90%. c. the incidence of grade 5 toxicity at the two highest dose-cohorts was 10%. d. primary tumor up to 7 cm were treated.

B Medically inoperable patients, predominantly elderly, were included in the study. The *MTD in the phase I data was 12Gy/fr x 5 fractions*. The *2-year local control rates* for the two highest cohorts (11.5 Gy and 12 Gy) were *89.4%* (90% CI: 81.6-97.4%) and *87.7%* (90% CI: 78.3-97%), respectively. Grade 5 toxicity attributed to SBRT was seen in 3 out of 71 patients treated at these two cohorts. *T1-2 (< 5 cm) tumors* were treated on this study. RTOG 0813: Grade 5 hemoptysis seen at multiple dose levels (10.5, 11.5, and 12 Gy), 2 G5 in 11.5 Gy cohort, 1 G5 in 12 Gy (pulm hemorrhage) -7.2% risk of dose limiting toxicity at highest dose level -0/8 grade 3+ toxicity at 50 Gy/5 fx

What is the mechanism of action for nivolumab? (A) Anti-angiogenic agent (B) Anti-PD-1 receptor antibody (C) BRAF V600E kinase inhibitor (D) Receptor tyrosine kinase inhibitor

B Nivolumab is a humanized monoclonal antibody that binds to the PD-1 receptor and blocks its interaction with PD-L1 and PD-L2. It was recently approved by FDA to treat patients with advanced NSCLC refractory to first line platinum-based chemotherapy. In a *phase-III trial (CHECKMATE 057), use of nivolumab was associated with improved survival (1-yr OS 51% vs 39%, median survival 12.2 m vs 9.4 m, p = 0.002)* when compared to single agent docetaxel, regardless of PD-L1 status.

Which chemotherapy regimen is the MOST appropriate 1st line therapy for an epithelial mesothelioma? (A) Cisplatin and Etoposide (B) Cisplatin and Pemetrexed (C) Carboplatin and Paclitaxel (D) Carboplatin and Gemcitibine

B Pemetrexed Trial(1999-2001) - Cisplatin vs. Cisplatin + Pemetrexed -Randomized. 456 patients with chemo-naive and inoperable mesothelioma. Arm 1) cisplatin 75 mg/m2 vs. Arm 2 cisplatin 75 mg/m2 + pemetrexed 500 mg/m2 2003"Phase III study of pemetrexed in combination with cisplatin versus cisplatin alone in patients with malignant pleural mesothelioma." (Vogelzang NJ, J Clin Oncol. 2003 Jul 15;21(14):2636-44.) -Outcome: *3 months increase of median OS* cisplatin 9months vs cisplatin/pemetrexed 12months (SS); time-to-progression 3.9 months vs. 5.7 months (SS), also improved lung function -Conclusion: Pemetrexed + cisplatin + vitamin supplements resulted in superior outcome compared with cisplatin alone

Regarding RTOG 0915 (Videtic) which compared two SBRT fractionation regimens (34 Gy in 1 Fx vs 48 Gy in 4 Fx), the single fraction treatment demonstrated: (A) a worse primary tumor control at 1 year. (B) a similar grade 3 or higher events at 1 year. (C) increased complications in the central lung tumors. (D) an improved abscopal effect by reducing distant metastases.

B RTOG 0915 enrolled medically inoperable patients *with peripheral* stage I non-small cell lung cancers only. 34 Gy in one fraction demonstrated *similar primary tumor control and protocol-specified adverse events as 48 Gy in 4 fractions*. This study was designed expecting to have similar local control while the arm with the least side effects was to be compared to 54 Gy in 3 fractions from RTOG 0236. The primary tumor control rates were 97% in the single fraction arm vs 92.7% in the four-fraction arm RTOG 0915: 34 Gy/1 Versus 48 Gy/4 · No difference in protocol specified toxicity: 10.3% in 34 Gy arm v 13.3% in 48 Gy arm · Median f/u 30 months · Grade 2 toxicities: fatigue 10% v 0% (34 Gy v 48 Gy), MSK disorders (8% v 0%), injury including fracture (8% v 2%), respiratory disorders (13% v 4%) · Similar trend for any toxicity (grade 1-5) · 2018 Refresher speaker comments that the 48/4 arm appears to look better in most of the curves despite not SS Videtic GM et al. Int J Radiat Oncol Biol Phys. 2015 Nov 15;93(4):757-64

What was the 5-year OS rate in the SWOG 9416/INT0160 trial for superior sulcus tumors? (A) 32% (B) 44% (C) 56% (D) 68%

B Superior Sulcus Tumors -If *operable*, concurrent chemo-RT (45Gy) → surgery → chemo (preferred) -If *marginally resectable*, concurrent chemo-RT (45Gy) → restage→ if no progression →surgery →chemo (INT0160) -If *unresectable* (initially or after restaging), *complete definitive chemo-RT (63-66Gy) → durvalumab (PD-L1 monoclonal antibody)* The *SWOG 9416/INT 0160* was a phase II study examining *neoadjuvant chemotherapy and radiation for patients with superior sulcus tumors*. Eligible stages were *T3-4N0-1*. Patients were treated to *45 Gy with concurrent cisplatin and etoposide*. *The rate of complete resection was 76%.* *pCR or minimal microscopic disease in 56%* *5 year overall survival was 44%.*

What is the MOST common pattern of relapse in patients presenting with superior sulcus tumors (T3-4 N0-1) treated with induction chemoRT followed by surgical resection? (A) Local recurrence (B) Distant metastases (C) Nodal recurrence within the mediastinum (D) Nodal recurrence within the supraclavicular fossa

B The *pattern of recurrence in the Intergroup study* was distant (non-brain) only (33%), brain only (33%), local only (17%), local + distant (12%). Despite the extensive local disease, *76% of patients underwent complete resection with pathological CR* or *minimal microscopic disease seen in 56%* of the resection specimens, resulting in *low local recurrence rates*. In appropriately staged patients with mediastinoscopy (or EBUS + PET- CT), recurrence in nodal regions outside of CTV remains low.

Regarding RTOG 0617, which evaluated (1) the benefit of an additional 14 Gy to the standard dose of 60 Gy and (2) the benefit of the addition of cetuximab to the chemotherapy? (A) No difference in OS was seen between all four treatment arms. (B) There was an OS benefit in the 60 Gy arm with either chemotherapy combination. (C) There was an OS benefit in the 74 Gy arm with either chemotherapy combination. (D) The OS benefit in the 74 Gy arm was limited to chemotherapy without cetuximab.

B The RTOG 0617 trial was a phase III study with a double randomization comparing (1) 60 vs. 74 Gy, and chemotherapy consisting of carbo/paclitaxel alone vs. carbo/paclitaxel with cetuximab. The median survival times and 18-month OS rates for the 60 Gy and 74 Gy arms were 28.7 vs 19.5 months, and 66.9% vs 53.9% respectively (p < 0.001). Unfortunately, and surprisingly, not only was there no benefit to escalating doses to 74 Gy, this arm did statistically worse.), may be related to higher doses to the heart or lower quality of radiation at higher dose on secondary analysis

What is the optimal management for a medically inoperable patient with a peripherally located 4.2 cm adenocarcinoma of the upper lobe with no evidence of metastatic disease? (A) SBRT using a single fraction of 34 Gy (B) SBRT using 3 fractions of 18 Gy (C) SBRT using 4 fractions of 12 Gy followed by adjuvant chemotherapy (D) Conventionally fractionated EBRT to 60 Gy with concurrent chemotherapy

B The appropriate treatment approach for a patient with a larger (> 3 cm) peripheral tumor can still be SBRT. -*Timmerman's initial phase I* (Chest 2003) using a 3 fraction scheme *included tumors up to 7 cm*, *Timmerman NEJM 2010. Videtic 8 reported a follow up study (RTOG 0915 - ASTRO 2014) comparing 2 less aggressive schemes *(34 Gy in 1 fx OR 12 Gy x 4)* although this study was tested in *tumors < 5 cm*. RTOG 06-18. SBRT for Operable Early-Stage Lung CA. Timmerman, et al. JAMA Oncology, 2018. - Single arm, Phase 2, enrolled patients 2007-10 with median f/u of 48.1 months. T1-T2, N0M0, NSCLC £ 5 cm, FEV1 and DLCO ³ 35% predicted. - SBRT dose was 54 Gy in 3 fx (18 Gy/fx) over 1.5-2.0 weeks. - 33 accrued, 26 evaluable. Only 1 patient with primary tumor recurrence. 4-year primary tumor control and local control rate 96%. 4 year DFS 57% and OS 56%, median OS 55.2 months. - SBRT appears to be associated with a high rate of primary tumor control, low treatment-related morbidity, and infrequent need for surgical salvage in patients with operable early-stage lung cancer *34 Gy is not an optimal approach* for this patient since only a *small percentage had tumors > 3 cm* to test its safety and efficacy plus the follow up for this trial is still very short. Currently the standard dose is 18Gy x3. The plan from the RTOG, NRG, is to compare 18Gy x3 with 34Gy in 1. *Most experts would consider 12 Gy x 4 to be equal to 18 Gy x3* although control rates may be lower for tumors > 3 cm based retrospective data. *The use of chemotherapy has not been tested* in this situation although some would consider it logical*based on the benefit seen in the postoperative situation especially since the patients are typically not surgically staged (ANITA). Although this patient population may find harm in the addition of chemotherapy (LACE meta-analysis)

A recent multi-institutional analysis of brain metastases from EGFR-mutant NSCLC which examined sequencing of SRS, WBRT, and EGFR-TKI therapy, demonstrated: a. upfront EGFR-TKI preserves neurocognition without a detriment in OS. b. SRS followed by EGFR-TKI resulted in the longest OS. c. upfront WBRT was inferior to upfront EGFR-TKI. d. upfront WBRT was superior to upfront SRS.

B The median OS for the SRS, WBRT, and EGFR-TKI cohorts was 46, 30, and 25 months, respectively. *Upfront SRS had the best survival outcomes* with improved survival in both favorable and unfavorable patient groups, when stratified by ds-GPA. Despite having more patients with less favorable prognosis, even on multivariate analysis *upfront WBRT had improved survival over upfront EGFR-TKI* (30 vs 25 months, median OS, p < 0.039). *Deferring RT was associated with inferior OS.*

Regarding adjuvant therapy for a NSCLC patient with a 6 cm adenocarcinoma with 3 involved hilar lymph nodes with tumor 5 mm from the bronchial margin: (A) no further therapy recommended. (B) post-operative RT is recommended if adjuvant chemotherapy refused. (C) post-operative concurrent chemoRT. (D) a completion pneumonectomy is recommended because of close margins.

B The recommended adjuvant treatment for this patient is *chemotherapy alone*. While PORT is currently recommended only for patients with *positive mediastinal disease (pN2)*, analysis of PORT delivered in the *ANITA trial demonstrated benefit of PORT in pN1 patients who did not receive adjuvant chemotherapy*

The tumor PD-L1 expression enrollment requirement in the KEYNOTE-010 phase 2/3 randomized study for previously treated advanced NSCLC (pembrolizumab vs docetaxel) was: a. No requirement. b. 1%. c. 5%. d. 50%.

B To be eligible for the *Keynote-010 study*, patients with previously treated advanced NSCLC were required to have at *least 1% tumor cells with PD-L1 expression*. In the total patient population, *median OS in the two pembrolizumab dose arms was significantly longer than docetaxel*. In a *planned subset analysis* of patients with expression of *PD-L1 on 50%* of the tumor cells, *median OS and PFS* in the two pembrolizumab dose arms was significantly longer than docetaxel. In *KEYNOTE-024 phase 3 trial*, of untreated advanced NSCLC with *PD-L1 expression on at least 50%* of tumor cells and *no EGFR mutation or ALK translocation* treated with *pembrolizumab (200 mg every 3 weeks)* or s choice of *platinum-based chemotherapy*. *Crossover* from the chemotherapy group to the pembrolizumab group was permitted in *disease progression*.

Separate tumors in the same lobe are considered to signify what AJCC stage of disease? (A) T2 (B) T3 (C) T4 (D) M1a

B Two tumor nodules in the *same lobe are considered T3 disease*. Two tumor nodules in *separate but ipsilateral lobes* are *T4 disease*, A separate tumor nodule in a *contralateral lobe* is considered *M1a disease*.

What is the minimum biologically effective dose (Gy10) to achieve a LC rate of at least 90% for SBRT in early stage NSCLC? (A) 75 (B) 100 (C) 125 (D) 150

B Zhang et al. explored the optimal biological effective dose (BED) range for the treatment of patients with Stage I non-small cell lung cancer with Stereotactic Body Radiation Therapy (SBRT). According to the quartile of the studies included into this meta-analysis, *BED was divided into four dose groups:* -low (<83.2 Gy), -medium (83.2-106 Gy), -medium to high (106-146 Gy), -high (>146 Gy). Corrected 3-year CSS in the medium (79.5%), medium to high (80.6%), and high groups (90.0%) were higher than in the low group (70.1%, p = 0.016, 0.018, 0.001, respectively). The biological effective dose (BED10) used for SABR treatment should be *at least 100 Gy* (grade B recommendation).

Following surgery for lung cancer, a patient was found to have a 4.6 cm adenocarcinoma, a second 1 cm focus in the same lobe, and a level 10 lymph node demonstrating metastatic disease. What is the correct T and N stage (AJCC8th Ed) for this patient with M0? (A) T2N1 (stageIIA) (B) T3N1 (stage IIIA) (C) T3N2 (stage IIIA) (D) T4N2 (stage IIIB)

B T1-Tumor ≤3 cm in greatest dimension, surrounded by lung or visceral pleura, without bronchoscopic evidence of invasion more proximal than the lobar bronchus (i.e., not in the main bronchus) T1mi-Minimally invasive adenocarcinoma: adenocarcinoma (≤3 cm in greatest dimension) with a predominantly lepidic pattern and ≤5 mm invasion in greatest dimension -T1a-Tumor ≤1 cm in greatest dimension. A superficial, spreading tumor of any size whose invasive component is limited to the bronchial wall and may extend proximal to the main bronchus also is classified as T1a, but these tumors are uncommon. -T1b-Tumor >1 cm but ≤2 cm in greatest dimension -T1c-Tumor >2 cm but ≤3 cm in greatest dimension T2-Tumor >3 cm but ≤5 cm or having any of the following features: • Involves the main bronchus regardless of distance to the carina, but without involvement of the carina • Invades visceral pleura (PL1 or PL2) • Associated with atelectasis or obstructive pneumonitis that extends to the hilar region, involving part or all of the lung T2 tumors with these features are classified as T2a if ≤4 cm or if the size cannot be determined and T2b if >4 cm but ≤5 cm. -T2a-Tumor >3 cm but ≤4 cm in greatest dimension -T2b-Tumor >4 cm but ≤5 cm in greatest dimension T3-Tumor >5 cm but ≤7 cm in greatest dimension or directly invading any of the following: parietal pleura (PL3), chest wall (including superior sulcus tumors), phrenic nerve, parietal pericardium; or separate tumor nodule(s) in the same lobe as the primary T4-Tumor >7 cm or tumor of any size invading one or more of the following: diaphragm, mediastinum, heart, great vessels, trachea, recurrent laryngeal nerve, esophagus, vertebral body, or carina; separate tumor nodule(s) in an ipsilateral lobe different from that of the primary N1-Metastasis in ipsilateral peribronchial and/or ipsilateral hilar lymph nodes and intrapulmonary nodes, including involvement by direct extension N2-Metastasis in ipsilateral mediastinal and/or subcarinal lymph node(s) N3-Metastasis in contralateral mediastinal, contralateral hilar, ipsilateral or contralateral scalene, or supraclavicular lymph node(s) pM1a-Separate tumor nodule(s) in a contralateral lobe; tumor with pleural or pericardial nodules or malignant pleural or pericardial effusion, microscopically confirmed. Most pleural (pericardial) effusions with lung cancer are a result of the tumor. In a few patients, however, multiple microscopic examinations of pleural (pericardial) fluid are negative for tumor, and the fluid is nonbloody and not an exudate. If these elements and clinical judgment dictate that the effusion is not related to the tumor, the effusion should be excluded as a staging descriptor. pM1b-Single extrathoracic metastasis in a single organ (including involvement of a single nonregional node), microscopically confirmed pM1c-Multiple extrathoracic metastases in a single organ or in multiple organs, microscopically confirmed

What is the benefit of PCI for a patient with extensive SCLC who had a good response to chemotherapy? (A) Reduction in the incidence of symptomatic brain metastases, but no effect on OS (B) Reduction in the incidence of symptomatic brain metastases, and an improvement in OS (C) Reduction in the incidence of asymptomatic brain metastases, and an improvement in OS (D) No reduction in the incidence of symptomatic brain metastases, but an improvement in OS

B The role of prophylactic cranial irradiation was evaluated by a randomized trial conducted in 186 patients with extensive small cell lung cancer who had had a response to chemotherapy. This study showed that prophylactic cranial irradiation *reduces the incidence of symptomatic brain metastases (HR0.27; 95% CI: 0.16-0.44, p < 0.001) and prolongs disease-free (14.7 weeks vs 12.0 weeks, p = 0.02) and overall survival (6.7 vs 5.4 months, p = 0.003)* Not only did they show a decrease in CNS metastases but also an *improvement of overall survival at 1 year (27% versus 13%).* The cumulative risk of brain metastases at 1 year was 40.4% in the observation arm and 14.6% in the therapy arm. Patients in *this study did not have routine CNS imaging*. brain *CT or MRI was done only if patients had symptoms of metastases.No survival benefit in subsequent Japanese Trial that used MRI screening but inferior chemotherapy.*

After completion of concurrent chemoRT with weekly paclitaxel (45-50 mg/m2) and carboplatin (AUC 2) for unresectable, stage III NSCLC, the next step is: a. PCI. b. consolidation durvalumab (10 mg/kg) every 2 weeks for 12 months. c. 2 cycles of paclitaxel (200 mg/m2) and carboplatin (AUC 6). d. consolidation nivolumab (3 mg/kg) every 2 weeks for 12 months

B or C It is *not a standard of care to offer PCI* for patients with stage III NSCLC after chemoradiation since randomized trials did *not demonstrate a survival benefit*. After completion of concurrent chemoRT with weekly paclitaxel (45-50 mg/m2) and carboplatin (AUC 2) the *next step is consolidation 2 cycles of paclitaxel (200 mg/m2) and carboplatin (AUC 6)*. Once this therapy is completed, based on the results from the recently published phase-III randomized trial (Antonia et al, NEJM 2017), patients with *PS 0-1* and *no disease progression* can be treated with *durvalumab (anti-PD L1 antibody) (10 mg/kg) every 2 weeks for 12 months or until progression.* Consolidation nivolumab or pembrolizumab is not currently recommended for non-metastatic advanced NSCLC after completion of chemoRT.

Compared to conventionally fractionated radiation (60 Gy) for unresectable stage I NSCLC, SBRT: (A) has no significant benefit in OS or local control. (B) is better in both local control and OS. (C) is better in local control but not in OS. (D) is better in OS but not in local control OS.

B. Conventionally fractionated radiation has worse local control and by retrospective reviews the comparison of overall survival is worse than most prospective or retrospective evaluations of overall survival, even in unresectable NSCLC.

In a patient with limited stage SCLC with a complete response after chemotherapy and thoracic irradiation, PCI: (A) has no benefit. (B) increases 3-year OS by about 5%. (C) improves DFS, but not OS. (D) has unacceptable risk of complications.

B. PCI given to patients with small-cell lung cancer in complete remission *decreases the risk of brain metastasis threefold* without a significant increase in complications. PCI improves both *overall survival and disease-free survival* among patients with small-cell lung cancer in *complete remission*. *PCI is associated with a 5.4 percent increase in the rate of survival at three years (15.3 percent in the control group vs. 20.7 percent in the treatment group).*

Which of the following radiation fractionations was the experimental arm in the phase III study by Turrisi (1999) for limited stage SCLC? (A) 37.5 Gy in 2.5 Gy fractions daily (B) 45 Gy in 1.5 Gy fractions twice daily (C) 45 Gy in 1.8 Gy fractions daily (D) 66 Gy in 1.2 Gy fractions twice daily

B. In both groups, the total dose of thoracic radiotherapy was *45 Gy for each patient*. Patients receiving *once-daily therapy received 1.8 Gy daily in 25 treatments* over a period of five weeks. *Accelerated twice-daily thoracic radiotherapy involved the administration administration of 1.5 Gy in 30 treatments over a period of three weeks*. In both groups, thoracic radiotherapy began *concurrently with the first cycle of chemotherapy*. The target volume for thoracic radiotherapy, which was similar in both groups, included the gross tumor, as defined by the chest CT scan, and the bilateral mediastinal and ipsilateral hilar lymph nodes. Irradiation of uninvolved supraclavicular fossae was forbidden. After a median follow-up of almost 8 years, the median survival was *19 months for the once-daily group and 23 months for the twice-daily group*. The survival rates for patients receiving once-daily radiotherapy were *41 percent at two years and 16 percent at five years*. For patients receiving twice-daily radiotherapy, the survival rates were *47 percent at two years and 26 percent at five years*. *Grade 3 esophagitis was significantly more frequent* with twice-daily thoracic radiotherapy, occurring in *27 percent of patients, as compared with 11 percent* in the once-daily group (P<0.001).

What was the key finding of RTOG 0617, a phase III trial exploring high dose (74 Gy) chemoradiation vs. standard dose (60 Gy) chemoradiation (+/- cetuximab) for patients with locally advanced NSCLC? (A) Local control was superior for the 74 Gy arm (B) OS was superior for the 60 Gy arm (C) OS was superior for the 74 Gy arm (D) Equivalent treatment related toxicities in both arms

B. In this pivotal trial, patients were randomly assigned to high dose (74 Gy) radiation vs. standard dose (60 Gy) chemoradiation, and then secondarily randomized to +/- cetuximab. Early results showed a paradoxical improvement in local control and overall survival for the 60 Gy arm. Chemotherapy was weekly carbo/taxol. No benefit was seen with the addition of cetuximab. *RTOG 0617* - Standard-dose versus high-dose conformal radiotherapy with concurrent and consolidation carboplatin plus paclitaxel with or without cetuximab for patients with stage IIIA or IIIB non-small-cell lung cancer (RTOG 0617): a randomized, two-by-two factorial phase 3 study -464 patients with stage III nonsmall-cell lung cancer (NSCLC)compared high-dose (74 Gy in 37 fx) with standard-dose (60 Gy in 20 fx) radiation. All patients also received chemotherapy with paclitaxel and carboplatin. -*High dose RT (74 Gy) a/w shorter survival and an increase risk of death compared with conventional RT(median OS 20 vs 29 months)*, may be related to higher doses to the heart or lower quality of radiation at higher dose on secondary analysis

According to the AJCC 8th edition staging manual for lung cancer, which of the following is classified as a T3 tumor? (A) Involves the main bronchus regardless of distance to the carina, but without involvement of the carina. (B) Tumor > 5 cm but < or equal to 7cm in greatest dimension (C) Separate tumor nodule(s) in the same lung, different lobe (D) Associated with atelectasis that extends to the hilar region

B. T1-Tumor ≤3 cm in greatest dimension, surrounded by lung or visceral pleura, without bronchoscopic evidence of invasion more proximal than the lobar bronchus (i.e., not in the main bronchus) T2-Tumor >3 cm but ≤5 cm or • Involves the main bronchus regardless of distance to the carina, but without involvement of the carina • Invades visceral pleura (PL1 or PL2) • Associated with atelectasis or obstructive pneumonitis that extends to the hilar region, involving part or all of the lung *T3-Tumor >5 cm but ≤7 cm in greatest dimension* or directly invading any of the following: *parietal pleura (PL3)*, *chest wall (including superior sulcus tumors)*, *phrenic nerve* *parietal pericardium*; *Separate tumor nodule(s) in the same lobe as the primary* T4-Tumor >7 cm or tumor of any size invading one or more of the following: diaphragm, mediastinum, heart, great vessels, trachea, recurrent laryngeal nerve, esophagus, vertebral body, or carina; separate tumor nodule(s) in an ipsilateral lobe different from that of the primary

What is the biological effective dose (BED) when 48 Gy of SBRT is given in 4 fractions for stage I NSCLC? (A) 72 Gy (B) 84 Gy (C) 105.6 Gy (D) 124.8 Gy

C (BED) for a given schedule against which the potency another can be compared. BED=nd(1+d/αβ radio) -dose, d, represents individual fraction size in Gy, -n=Number of fraction sizes -α/β ratio of 10 is assumed by the RTOG trials. BED=nd(1+d/αβ radio)=12*4(1+12/10)=105.6 -60 Gy in 30 fractions would have BED of 72 Gy, -70 Gy in 35 fractions equal BED of 84 Gy. -48 Gy in 3 fractions would equal BED of 124.8 Gy.

Regarding the role of elective nodal radiation tested in a randomized trial for stage III NSCLC treated with concurrent chemoRT (Yuan et al. 2007): (A) overall response rate was the same. (B) the RT pneumonitis rate was the same. (C) the rate of LC was better in the involved field arm. (D) OS was better in the involved field arm.

C A total of 200 eligible patients with inoperable stage III non-small cell lung cancer (NSCLC) were treated with concurrent chemoradiotherapy and randomized into either an involved-field irradiation (IFI) or elective nodal irradiation (ENI) arm. A total of 4 to 6 cycles of cisplatin-based chemotherapy were delivered, and concurrent radiotherapy was started after the second cycle of chemotherapy. Three-dimensional conformal radiotherapy was delivered in once-daily fractions of 1.8 to 2 Gy to 68 to 74 Gy for IFI or 60 to 64 Gy for ENI. *Patients in the IFI arm achieved better overall response rate (90% vs. 79%, P = 0.032) and better 5-years local control rate (51% vs.36%, P = 0.032) than those in the ENI arm.* *The radiation pneumonitis rate in patients with IFl was lower than in patients with ENI (17% vs. 29%, P = 0.044)*, and similar trends appeared in the radiation esophagitis, myelosuppression, and radiation pericarditis between 2 study arms, although not significantly. The 1-, 2-, and 5-year survival rates were 60.4%, 25.6%, and 18.3% for the ENI arm and 69.9%, 39.4%, and 25.1% for the IFI arm, respectively. *Only the 2-year survival rates were statistically significant (P = 0.048).* IFI arm achieved better overall response and local control than ENI arm, and it allowed a dose of 68 to 74 Gy to be safely administered to patients with inoperable stage III NSCLC.

What approximate percentage of SCLC patients present with extensive disease in the U.S.? (A) <20% (B) 30-40% (C) 60-70% (D) >80

C Approximately 35,000 Americans are diagnosed with small cell lung cancer annually. The incidence of *extensive disease is 60-70%*. This percentage of patients with ED has *increased over the last 20 years*, and this is at least partially due to *stage migration secondary to routine use of CT scans, brain MRIs, and PET.* *PET alone upstages 8%* of patients diagnosed with limited disease based on conventional staging (Bradley; J Clin Oncol. 22(16):3248-54, 2004; Niho; Lung Cancer. 57(3): 328-332007).

For a patient with extensive stage SCLC receiving cisplatin/etoposide, the expected response rate (RR) and the median OS associated with this regimen is: (A) RR of 30-50% and median OS of 4-5 months. (B) RR of 30-50% and median OS of 7-9 months. (C) RR of 60-80% and median OS of 7-9 months. (D) RR of 60-80% and median OS of 18-24 months.

C Chemotherapy is the standard treatment for extensive-disease small cell lung cancer. The combination of *cisplatin and etoposide (EP) or carboplatin and etoposide (EC)* up to 4-6 cycles remains the *most widely used* regimen, with an *RR ranging from 60% to 80%*. However, the median *OS is still 7- 9 months*, and only 2% of patients survive for 5 years. *Addition of consolidation RT* to responders with PCI in the Slotman paper showed a *2-year OS improvement was noted (13% vs. 3%, p=0.004).*

Based on a large multi institutional experience treating patients with concurrent chemoRT for advanced NSCLC (Palma), which dosimetric factor BEST predicted grade 2 or higher esophagitis? (A) V20 of the Esophagus (B) V30 of the Esophagus (C) V60 of the Esophagus (D) Mean Dose to the Esophagus

C Esophagitis is the primary side effect that occurs during radiotherapy and this article is the 1st large data set to evaluate multiple variables is a multi-institutional setting. They *systematically evaluated DVH of the esophagus from V5 through V70 by 5 Gy increments* as well as the mean, max dose total dose and fractionation. The best model was found to be in the *high dose volumes such as V50 and V60 which predicted both grade 2 and 3 events*.

Which method is used for detection of the oncogenic EML4-ALK translocation in patients with lung adenocarcinoma? (A) DNA sequencing (B) Polymerase chain reaction (C) Fluorescent in situ hybridization (D) Immunofluorescence microscopy

C Fluorescent in-situ hybridization (FISH) is used clinically to identify a chromosomal translocation resulting in an aberrant fusion gene that encodes a constitutively active ALK. It took only 3 years from discovery of this alteration in lung cancers to the report of dramatic clinical responses upon treatment with the small molecule inhibitor crizotinib, leading to FDA approval in 2011. Whether these alterations predominate in stage IV disease remains to be determined. This is an important example of molecular profiling of cancers impacting therapy. References: Shaw et al. J Clin Oncol 2011; 27:4247-425

Following surgery for a large cell carcinoma of the left lung, pathology demonstrated a 2.5 cm primary cancer with negative margins where there were four involved level 11 lymph nodes and 1 AP window lymph node that demonstrated metastatic disease. What is the recommended treatment following surgery? (A) Cisplatin based doublet x 4 cycles (B) Mediastinal RT followed by cisplatin based doublet x 4 cycles (C) Cisplatin based doublet x 4 cycles followed by mediastinal RT (D) Concurrent chemoRT to the mediastinum with carboplatin + paclitaxel

C For patients with *pathologic stage IIIA disease which is evidence by the AP window lymph node, mediastinal radiation is recommended (NCCN guidelines)* based on an abundance of data based on large national registries (SEER- Lally, and Robinson -NCDB). *Although concurrent chemotherapy has been shown to be safe and effective, it has not been show to improve survival in the adjuvant setting.* The preference is to follow the *sequential approach based on the large European Trials (Douillard).*

Based on level 1 evidence, what would be recommended following a completely resected 5.1 cm intermediate grade adenocarcinoma without evidence of lymphadenopathy? (A) Observation (B) Sequential postoperative chemoRT (C) Discuss the role of adjuvant chemotherapy (D) Adjuvant chemotherapy using a cisplatin based doublet

C In *CALGB 9633* in patients with stage IB (T2N0) that are completely resected were randomized to adjuvant chemo vs. not, and the *trial was negative* for the benefit of adjuvant chemo *except for the large tumors >=4.0 cm* in an unplanned subset analysis. In the subset of patients with larger tumors, they found an improvement in OS with the use of adjuvant chemo. While *not level 1 evidence*, this suggests a possible consideration for the role of adjuvant chemotherapy, which needs to be discussed with the patient

The EORTC trial demonstrated that PCI for patients with extensive stage SCLC who had a response to chemotherapy: (A) increased median OS by 12 weeks. (B) had a clinically significant effect on performance status. (C) decreased the cumulative risk of brain metastases at 1 year by 15%. (D) prolonged the DFS, but not OS.

C In the Slotman study Prophylactic cranial irradiation reduces the incidence of symptomatic brain metastases and prolongs disease-free and overall survival for patients with extensive-stage small-cell lung cancer who had a response to systemic chemotherapy. PCI-Extensive Stage 1. Slotman et al (2007). Phase III trial. 286 patients randomized to +/- PCI after chemo. Fractionation schedules: 20/5fx, 30/10fx, 30/12fx, 25/10fx. a. 1yr OS *27.1% vs 13.3% (SS p = 0.003)*. b. Cumulative risk of brain metastases within 1 year was *14.6% in the irradiation group (95% CI, 8.3 to 20.9) and 40.4% in the control group (95% CI, 32.1 to 48.6)*. c. Side effects, but no clinically significant effect on global health status. d. Limitation: brain imaging not part of standard staging/follow up unless symptomatic 2. Takahashi et al (2017). Randomized. 224 patients; 163 patients reached futility. ED-SCLC w/any response to platinum doublet chemo -/+ PCI at 25 Gy/10 fractions. a. MRI required prior to enrollment b. Median OS not significantly different 11.6 vs 13.7 m for PCI v Obs (P=0.09). i. 2-year OS 15% in PCI vs 18.8% in obs ii. PCI reduced risk of BM 48% vs 69% (P<0.001) iii. PFS comparable, median 2.3 vs 2.4 m (NS) iv. Grade 3+ toxicities: Anorexia (6% PCI v 2%), malaise (3% v 1%), muscle weakness in LE (<1% v 5%) v. MMSE scores NS in difference at baseline, 12 m, 24 m vi. RT for BM in 46% of PCI group, 83% of obs group

In the retrospective review of NSCLC patients treated with definitive 3DCRT by Graham et al., which of the following factors was found to be the only predictor of Grade 2 or more radiation pneumonitis? a. Sequential or concurrent chemotherapy b. Mean lung dose c. Lung volume exceeding 20 Gy (V20) d. Location of the primary tumor (upper vs lower lobes)

C In this retrospective review of 99 patients treated with definitive 3DCRT, the lung V20, Veff and mean dose, and location of primary tumor (upper vs lower lobe) predicted for Grade ≥2 radiation pneumonitis on univariate analysis. *A lung V20 > 40%* was associated with an actuarial incidence of *Grade ≥ 2 pneumonitis of 36%* while a lung *V20 < 22% resulted in no incidences of Grade ≥2 pneumonitis*. On multivariate analysis, *only the lung V20 predicted for radiation pneumonitis (P=0.001)*.

The randomized trial by EORTC and RTOG comparing standard PCI dose to higher-dose in patients with limited-stage SCLC showed the higher dose of PCI: (A) improved CNS disease-free survival. (B) reduced overall mortality rates. (C) did not reduce brain metastases. (D) increased the rate of CNS toxicity.

C No significant reduction in the total incidence of brain metastases was observed after higher radiotherapy dose for PCI when compared with standard radiotherapy dose for PCI, but there was a significant increase in mortality. *PCI at 25 Gy should remain the standard of care in limited-stage SCLC.* References: LePechoux C, Dunant A, Senan S, et al. Standard-dose versus higher-dose prophylactic cranial irradiation (PCI) in patients with limited-stage small-cell lung cancer in complete remission after chemotherapy and thoracic radiotherapy (PCI 99-01, EORTC 22003-08004, RTOG 0212, and IFCT 99-01): a randomized clinical trial. The Lancet 2009; 10(5):467-474.

What are the corresponding nodal stations for a patient with a left upper lobe NSCLC and pathologic involvement of the ipsilateral hilar and lower paratracheal lymph nodes? (A) 10,7 (B) 12,5 (C) 10,4 (D) 12,2

C Nodal stations 10-14 are classified as Nl, nodal stations 1-9 are classified as N2 *Supraclavicular nodes* 1. Low cervical, supraclavicular and sternal notch nodes From the lower margin of the cricoid to the clavicles and the upper border of the manubrium. The midline of the trachea serves as border between 1R and 1L. *Superior Mediastinal Nodes 2-4* 2R Upper Paratracheal 2R nodes extend to the left lateral border of the trachea. From upper border of manubrium to the intersection of caudal margin of innominate (left brachiocephalic) vein with the trachea. 2L. Upper Paratracheal-From the upper border of manubrium to the superior border of aortic arch. 2L nodes are located to the left of the left lateral border of the trachea. 3A. Pre-vascular-These nodes are not adjacent to the trachea like the nodes in station 2, but they are anterior to the vessels. 3P. Pre-vertebral-Nodes not adjacent to the trachea like the nodes in station 2, but behind the esophagus, which is prevertebral. 4R. Lower Paratracheal From the intersection of the caudal margin of innominate (left brachiocephalic) vein with the trachea to the lower border of the azygos vein. 4R. nodes extend from the right to the left lateral border of the trachea. 4L. Lower Paratracheal-From the upper margin of the aortic arch to the upper rim of the left main pulmonary artery *Aortic Nodes 5-6* 5. Subaortic-These nodes are located in the AP window lateral to the ligamentum arteriosum. These nodes are not located between the aorta and the pulmonary trunk but lateral to these vessels. 6. Para-aortic-These are ascending aorta or phrenic nodes lying anterior and lateral to the ascending aorta and the aortic arch. *Inferior Mediastinal Nodes 7-9* 7. Subcarinal 8. Paraesophageal-Nodes below carina. 9. Pulmonary Ligament-Nodes lying within the pulmonary ligaments. *Hilar, Lobar and (sub)segmental Nodes 10-14-These are all N1-nodes.* 10. Hilar nodes-These include nodes adjacent to the main stem bronchus and hilar vessels. On the right they extend from the lower rim of the azygos vein to the interlobar region. On the left from the upper rim of the pulmonary artery to the interlobar region.

A 75-year-old presented with weight loss, progressive shortness of breath, and recent onset of delirium. Workup shows a large mediastinal mass and a sodium level of 116 mEq*/L. What is the next step in management? (A) Brain MRI (B) FDG-PET/CT (C) Hypertonic saline (D) Bronchoscopy and biopsy

C Patients with *SIADH and acute onset of symptomatic hyponatremia* should be managed with *slow infusion of hypertonic saline* and *not to exceed a correction of 1 to 2 nmol per L per hour*. All other choices are not appropriate for the acute management of the patient's hyponatremia which is the result of SIADH, likely related to small cell lung cancer.

According to the AJCC 8th edition staging manual for NSCLC, which of the following is classified as T4? (A) Chest wall invasion (B) Separate tumor nodule(s) in the same lobe (C) Separate tumor nodule(s) in a different ipsilateral lobe (D) Involvement of the main bronchus, ≥ 2 cm distal to the carina

C T4 -Separate tumor nodule(s) in an ipsilateral lobe different from that of the primary -Tumor >7 cm or tumor of any size invading one or more of the following: -diaphragm, -mediastinum, -heart, -great vessels, -trachea, -recurrent laryngeal nerve, -esophagus, -vertebral body -carina A&B-->T3: -chest wall (including superior sulcus tumors), -phrenic nerve, -parietal pericardium; or separate tumor nodule(s) in the same lobe as the primary. D-->T2: • Involves the main bronchus regardless of distance to the carina, but without involvement of the carina • Invades visceral pleura (PL1 or PL2) • Associated with atelectasis or obstructive pneumonitis that extends to the hilar region, involving part or all of the lung

What, if any, adjuvant therapy should be recommended based on level 1 evidence for a lung cancer patient with 3 cm adenocarcinoma with negative margins with involvement of one level 2 and one level 4 lymph node? (A) Concurrent chemoRT using weekly carboplatin and paclitaxel (B) Cisplatin and Pemetrexate (C) Cisplatin and Vinorelbine (D) Concurrent chemoRT using weekly Cisplatin

C The ANITA trial is an important trial that used Cisplatin in combination with Vinorelbine demonstrating a benefit in survival. Concurrent strategies have been overall neatie in the adjuvant setting have been shown to be safe and efficacious in phase II studies but have failed to demonstrate benefit a benefit in a phase 3 setting (Bradley, ECOG). Cisplatin Pemetrexate is commonly used in the adjuvant setting although the data is limited to the metastatic setting -N2 margins are negative, adjuvant chemo for 2-4 Cycles → mediastinal RT - Involved LN region +/- ipsilateral hilum +/- subcarinal LN region to 50.4 Gy depending on the extent of node dissection, number, bulk, and location of mediastinal disease and primary tumor -10-16Gy boost if extranodal extension -If positive margin (R1), favor initial post-op RT →adjuvant chemo or post op concurrent chemo-RT (e.g., carbo/taxol with RT). Limit field to area of +margin if N0-N1 disease (i.e. no elective mediastinal nodal coverage). -If gross residual disease (R2)→ favor concurrent chemo-RT ANITA: 840 pts with stage IB(T2aNo), II or IIIA. Assigned to either postop chemotherapy (vinorelbone/cisplatin) or observation. Co-Retrospective analysis of prospective data. Patients randomized to surgery +/- chemo, and PORT was for pN+ pts (but not required). 232/840 (28%) received RT 45-60 Gy (8% of N0 pts, 35% of N1, 52% of N2). ● pN2: benefit for PORT regardless of chemo arm; if no chemo (1.1 years vs. 1.9 years), if chemo (2.0 years vs. 3.9 years) ● Conclusion: Positive effect of PORT in pN2 patients, negative effect in pN1 patients who were treated with chemotherapy ● Suggests that the effect of PORT is influenced by the extent of nodal involvement and the use of adjuvant chemotherapy No good recent randomized data using modern techniques.

Regarding the role of surgery following chemoRT for stage III non-small cell lung cancer: (A) 60 Gy is required to demonstrate an improvement in OS with surgery. (B) surgery improves OS in all resectable patients based on level 1 evidence. (C) a lobectomy improves local control and OS based an unplanned subgroup analysis of the intergroup trial. (D) a pneumonectomy demonstrates a local control and OS for left side tumors based an unplanned subgroup analysis of the intergroup trial.

C The Intergroup experience published by Albain demonstrated a *significant improvement in local regional control with the addition of surgery following chemo radiotherapy over radiotherapy alone* although this *did not translate into a survival advantage* related to the *mortality of surgery* - primarily the right sided pneumonectomies. *An unplanned analysis of this data suggested that patients undergoing a lobectomy had less morbidity/mortality* and subsequently this group of patients had a survival advantage.

Which chemotherapy has been shown to improve OS when combined with Cisplatin for patients with malignant pleural mesothelioma? (A) Paclitaxel (B) Vinorelbine (C) Pemetrexed (D) Gemcitabine

C The addition of pemetrexed at 500 mg/m2 to 75 mg/m2 of cisplatin every 21 days improved median overall survival (12.1 months vs 9.3 months), median time to progression (5.7 months vs 3.9 months) and improved response rates (41% vs 17%) compared to cisplatin alone. 1. Pemetrexed Trial(1999-2001) - Cisplatin vs. Cisplatin + Pemetrexed -Randomized. 456 patients with chemo-naive and inoperable mesothelioma. Arm 1) cisplatin 75 mg/m2 vs. Arm 2 cisplatin 75 mg/m2 + pemetrexed 500 mg/m2 2003"Phase III study of pemetrexed in combination with cisplatin versus cisplatin alone in patients with malignant pleural mesothelioma." (Vogelzang NJ, J Clin Oncol. 2003 Jul 15;21(14):2636-44.) -Outcome: median OS cisplatin 9months vs cisplatin/pemetrexed 12months (SS); time-to-progression 3.9 months vs. 5.7 months (SS), also improved lung function -Conclusion: Pemetrexed + cisplatin + vitamin supplements resulted in superior outcome compared with cisplatin alone

In the Auperin meta-analysis of limited stage SCLC patients, PCI reduced the 3-year incidence of brain metastases from 58.6% to: (A) 13%. (B) 23%. (C) 33%. (D) 43%.

C The findings from individual-patient data meta-analysis of patients with SCLC in CR treated with or without PCI showed *improvement in 3-year survival from 15.3% to 20.7%* and reduced incidence of *brain metastases from 58.6% to 33.3%*, a *25% absolute difference in brain metastases.* Auperin (1999). Meta-analysis of (7) Phase III/randomized studies. 987 patients. Patients with complete remission. Extensive disease in 12-17%. Compared PCI to no PCI after CR achieved. a. Outcome: i. 3-year OSPCI+ 20.7% vs. PCI- 15.3% (absolute benefit 5.4%, SS). ii. 3-year LC 33% vs. 59% (SS). iii. DFS also improved. b. RT dose: larger doses led to greater decrease in risk of mets, but no impact on survival. c. Timing: decreased risk of mets with earlier administration after induction chemo <4-6months vs>6 months. d. Conclusion: PCI improves overall survival, DFS, and control of brain metastases. e. Critique: 4/7 trials had <100 patients, ~14% had extensive disease, dose-fractionation not uniform.

Concerning the CALGB 9633 study assessing the benefit of adjuvant chemotherapy for patients with resected T2N0 NSCLC, there was a benefit for: (A) all subgroups. (B) patients with visceral pleural invasion. (C) patients with tumors ≥ 4 cm. (D) patients with incompletely resected tumors.

C There was no difference in any outcomes overall, but only in unplanned subset analysis showing *survival benefit of >=4 cm tumors*. Grades 3 to 4 neutropenia were the predominant toxicity; there were no treatment-related deaths. *Survival was not significantly different* (hazard ratio [HR], 0.83; CI, 0.64 to 1.08; P = .12). However, *exploratory analysis demonstrated a significant survival difference in favor of adjuvant chemotherapy for patients who had *tumors > or = 4 cm in diameter* (HR, 0.69; CI, 0.48 to 0.99; P = .043). CONCLUSION: statistically significant survival advantage for patients who had tumors > or = 4 cm supports consideration of adjuvant *paclitaxel/carboplatin for stage IB patients who have large tumors*

Testing for EGFR mutations and ALK rearrangement is recommended for which of the following lung cancers? (A) Small cell carcinoma (B) Squamous cell carcinoma (C) Adenocarcinoma (D) Neuroendocrine carcinoma

C. Several biomarkers have been shown to be prognostic and predictive. Among these biomarkers, the evidence is strongest for EGFR, ERCC1, KRAS and ALK. Testing EGFR mutation and ALK gen rearrangements is recommended for patients with adenocarcinoma so that patients with these genetic abnormalities can receive effective targeted therapy such as *erlotinib (EGFR) and criztinib (ALK)*.

What is the BEST treatment option for a healthy patient with a limited stage SCLC? (A) Chemotherapy alone (B) Chemotherapy followed by thoracic irradiation (C) Concurrent chemotherapy and thoracic irradiation (D) Thoracic irradiation after complete response to chemotherapy

C. Although complete remission rates were not significantly different between the two arms, *progression-free survival* (P = .036) and *overall survival* (P = .008) were *superior in the early thoracic irradiation arm*. Patients in the late thoracic irradiation arm had a *higher risk of brain metastases* (P = .006). A small but significant improvement in 2- year OS for early radiotherapy versus late radiotherapy in limited stage small cell lung cancer was observed. Timing of radiation therapy 3 studies showing the benefit of concurrent ChemRt 1. Takada (2002)/Japanese Clinical Oncology Group. Randomized. 231 pts. All pts received 4 cycles of cisplatin 80 mg/m2 + etoposide 100 mg/m2 chemotherapy. Randomized to: Arm 1) Concurrent RT 45/30 BID starting day 2 vs. Arm 2) Sequential RT 45/30 BID based on pretreatment volumes. PCI 24/16 BID if CR/near CR a. Outcome: 5-year OS concurrent 23.7% vs. sequential 18.3% (NS). 5-year OS 24% vs 18% (NS). After adjustment, hazard ratio for death in concurrent arm 0.7 (SS). Brain mets concurrent 19% vs sequential 27% (no p). Concurrent RT had worse hematologic toxicityand worse severe esophagitis (9% vs. 4%). b. Conclusion: Suggestion (NS) for benefit of concurrent vs sequential chemo-RT. 2. Fried (2004). Meta-analyses of randomized controlled trials performed on L-SCLC. a. Improved survival for early concurrent (<9wks vs >9wks) integration of RT with platinum-based chemo. b. 5.2% better OS with early RT. 3. Jeremic (1997). RT week 1 vs week 6. Randomized. 107 patients treated by ACC Hfx RT (54 Gy in 1.5 Gy/fx BID) with concurrent carboplatin/etoposide (C/E), randomized to either: group 1. week 1-4 RT, group 2 or week 6-9 RT a. Median survival: 34 months vs. 26 months (p=0.052), 5-year OS 30% vs. 15% (p=0.03). Significantly higher LRFS for group 1. Toxicity comparable b. Conclusion: Initial thoracic concurrent ACC HFX RT with chemo better LC and survival than delayed RT.

Which of the following drugs selectively targets anaplastic lymphoma kinase (ALK) mutated NSCLC? (A) Gefitinib (B) Erlotinib (C) Lapatinib (D) Crizotinib

D *Crizotinib is an oral tyrosine kinase inhibitor targeting ALK rearranged NSCLC*. pretreated patients it was associated with a *57% overall response rate*, and a estimated *6 month progression free survival of 72%*. *Erlotinib and gefitinib* are *EGFR inhibitors*. *Lapatinib* is a *mixed EGFR and Her2 inhibitor*.

Regarding early stage NSCLC eligible for treatment with SBRT: (A) local control decreases if more than 5 fractions are given. (B) adjuvant chemotherapy improves DFS and OS following SBRT. (C) there is level 1 evidence that SBRT has equivalent OS in comparison to surgery. (D) a BED of 100 Gy is an important dose to achieve high local control rates > 90%.

D According to *Japanese data, if the BED is ≥100 Gy, then the 5-yr LC rate is 92% and the 5-yr OS is 71%*. However, if the *BED is <100 Gy, then the 5-yr LC rate is 57% and the 5-yr OS is 30%.* There is no data suggesting a benefit of adjuvant chemotherapy following SBRT. There are no phase III trials that have been reported to date comparing surgery and SBRT for early lung cancer. ROSEL AND STARS trials showed improved OS in operable patients, but theses trials failed to accrue.

What is the BEST treatment approach for a T3N0 malignant epithelioid mesothelioma? (A) Platinum-based chemotherapy (B) Extrapleural pneumonectomy alone (C) Platinum-based chemotherapy, followed by concurrent chemoRT (D) Induction chemotherapy, pleurectomy and decortication, followed by IMRT

D Andreas Rimner at *MSKCC in a multicentre phase II study* used pleural based IMRT following pleurectomy/decortication for localized mesothelioma is an appropriate and safe option for therapy. *Induction chemo, decortication and pleural-based IMRT is an excellent option* for patients without compromised (if not better) survival than patients treated historically with EPP and radiation therapy. *Results appear promising and may be safer and more effective than historical data with EPP followed by adjuvant radiation*.

Regarding surgical planning for patients with thymoma: (A) pericardial involvement is a contraindication for surgery. (B) maximal debulking should be aimed in unresectable patients. (C) CT-guided trans-pleural biopsy is recommended prior to surgery. (D) serum anti-acetylcholine receptor antibody levels should be obtained pre-surgery.

D Approximately *30-50% of patients with thymomas have myasthenia gravis*. Hence, before any surgical procedure, all patients suspected of having thymomas should have their serum antiacetylcholine receptor antibody levels measured. Surgical biopsy is not necessary if a resectable thymoma is strongly suspected clinic-radiologically. *If biopsy is performed, a transpleural approach is avoided to prevent tumor seeding.* *Unresectable patients are treated with induction therapy* followed by re-assessment for surgery. *Pericardial resection with graft reconstruction is attempted when feasible.*

What is the strongest predictor of survival for patients with IIIA NSCLC treated with neoadjuvant chemoradiation and resection? (A) Total dose of radiation (B) Intensity of chemotherapy (C) Primary tumor response (D) Pathologic nodal clearance

D IIIA NSCLC: *T1-2 N2 disease or T3-N1 and T4 N0-N1* Rationale: The strongest predictor of long- term survival was *mediastinal nodal clearance*, which had a *hazard ratio of 0.22*, (p=.0003) (Betticher et al., J Clin Oncol. 21(9): 1752-1759, 2003). Most importantly, in the phase III trial, INT 0139, nodal clearance predicted outcome. *Patients with N0 resection specimens demonstrated a 41% 5-year survival versus 24% for those with pN1-3 in the surgical specimen* (Albain et al. J Clin Onc. 23:16S: 7014, 2005). Clearly, studies of induction therapy (chemotherapy or chemoradiotherapy) followed by surgical resection have consistently demonstrated that *mediastinal nodal sterilization is a powerful predictor of outcome and can serve as a surrogate marker.*

In a secondary analysis of the KEYNOTE-001 study, use of previous radiotherapy for patients with NSCLC receiving pembrolizumab: a. increased anemia. b. increased esophagitis. c. increased pulmonary toxicity. d. improved PFS and OS.

D In patients who previously received any radiotherapy than in patients without previous radiotherapy use of pembrolizumab was *associated with longer PFS* (HR 0·56) *median PFS* 4·4 months vs 2·1 months and *longer OS* (HR 0·58); median overall survival 10·7 months vs 5·3 months Grade ≥ 3 pulmonary toxicity with or without thoracic RT was similar (17% vs 12%). Any pulmonary toxicity was 63% vs 40% (p = 0.058). Differences in anemia or esophagitis were not reported.

What is the contralateral lung dose constraint when using IMRT after extrapleural pneumonectomy? a. V20 <35% b. V20 <20 % c. Mean lung dose <15 Gy d. Mean lung dose <8.5 Gy

D In the early MDACC IMRT experience, *fatal pulmonary death was seen in ~10% of patients*. pulmonary related death with a *V20 of >7% was 42-fold higher* and *mean lung dose (MLD) of >8.5 Gy was 8.6-fold*. The authors concluded to keep *V20 < 7% and MLD < 8.5 Gy.* Constraint dose for Mesothelioma Lung <20% to receive >20Gy and Mean <8.5Gy Liver <30% to receive >30Gy Contralateral Kidney<20% to receive >15Gy Heart <50% to receive >45Gy Spinal Cord <10% to receive >45Gy with no potion receiving >50Gy Esophagus <30% to receive >55Gy

Which technique allows for biopsy of a level 6 mediastinal lymph node? (A) Endoscopic ultrasound (B) Cervical mediastinoscopy (C) Endobronchial ultrasound (D) Anterior (Chamberlin) mediastinoscopy

D It is helpful to know the differences between the various techniques for mediastinal lymph node staging. *Endobronchial ultrasound* can usually sample levels 2L, 2R, 4L, 4R, 7, and 10R and 10L *Endoscopic ultrasounds* can usually sample 4L, 4R, 7, 8, 9. *Cervical mediastinoscopy* can usually sample *2L, 2R, 4L, 4R, 7, and 10R and 10L* *Anterior mediastinoscopy* is very useful for 4L, 5, 6, and 7. It is *one of few techniques that can reach level 6.*

In a meta-analysis of randomized trials (Auperin et al., JCO 2010), the use of concurrent chemoRT (versus sequential chemoRT) in patients with locally advanced NSCLC demonstrated: a. increased acute grade 3-4 esophagitis. b. improved LC, but not DM or OS. c. improved LC and DM, but not OS d. improved LC and OS, but not DM.

D Meta-Analysis of Concomitant Versus Sequential Radiotherapy in Locally Advanced Non-Small Cell Lung Cancer (Auperin, JCO 2012) Concurrent chemoRT *increased OS by 4.5% at 5 years* 3-year OS 23.8% in concurrent chemoRT vs 18.1% in sequential *decreased locoregional progression (HR 0.77 (p= 0.01)*. *No effect* (HR of 1.04) was noted on *distant progression*. More Acute toxicity (myelosuppression and esophagitis). *Grade 3-4 esophagitis increased* from 4% to 18% with a HR of 4.9.

What is the proper management for a SCLC patient with symptomatic thoracic disease but has extensive stage disease based on a solitary asymptomatic brain metastasis? (A) Chemotherapy followed by PCI (B) WBRT plus SRS followed by chemotherapy (C) Definitive chemoRT to the thorax followed by PCI (D) Chemotherapy followed by RT to the thorax and brain, based on no evidence of progression

D Patients with extensive stage SCLC can be managed with *induction chemotherapy with cis or carboplatin / etoposide for 4 cycles*, and if there is *no evidence of disease progression*, to be consolidated with chest radiotherapy to 30 Gy in 10 fx (Slotman 2014) and whole brain radiation.* Given the patient is *asymptomatic from the disease in the brain* and the fact that *response rates in the brain are equal to response rates in the chest* with chemotherapy, *immediate radiation treatment to the brain is not necessary.*

Which of the following is NOT an adverse prognostic factor for patients with SCLC? (A) Continuation of smoking (B) Lower performance score (C) Elevated alkaline phosphatase level (D) Female gender

D Per NCCN: *Poor prognostic factors*: o Poor PS (3-4) o Extensive-stage disease o Weight loss (>5% in 6 months) o Markers associated with excessive bulk of disease (ie LDH) are the most important adverse prognostic factors *Favorable prognostic factors*:(in patients with extensive-stage disease) o Female gender o Age <70 o Normal LDH, creatinine levels o Single metastatic site

In the randomized Phase-II RTOG 0915 study comparing two SBRT schedules for medically inoperable patients with stage I peripheral NSCLC, the single fraction SBRT dose was: a. 21 Gy. b. 24 Gy. c. 30 Gy. d. 34 Gy.

D RTOG 0915 study randomized patients with medically inoperable, peripherally located, stage I NSCLC to receive either *34 Gy in 1 fraction* or *48 Gy in 4 fraction*. The incidence of *grade 3 or more adverse events* at 1 year were *10.3% and 13.3% respectively*. At 1 year, *primary tumor control rates* were *97% and 92.7%* respectively. RTOG 0915: 34 Gy/1 Versus 48 Gy/4 · No difference in protocol specified toxicity: 10.3% in 34 Gy arm v 13.3% in 48 Gy arm · Median f/u 30 months · Grade 2 toxicities: fatigue 10% v 0% (34 Gy v 48 Gy), MSK disorders (8% v 0%), injury including fracture (8% v 2%), respiratory disorders (13% v 4%) · Similar trend for any toxicity (grade 1-5) · 2018 Refresher speaker comments that the 48/4 arm appears to look better in most of the curves despite nott SS Videtic GM et al. Int J Radiat Oncol Biol Phys. 2015 Nov 15;93(4):757-64

In the phase III randomized study (CREST) evaluating the addition of 30 Gy of consolidative chest RT to chemotherapy and PCI for extensive stage SCLC: a. the primary endpoint of improved OS at 1 year was met. b. PCI had greater survival impact than the chest radiation. c. a doubling of grade 3-4 toxicities was observed. d. a 10% improvement in 2-yr OS was seen.

D Slotman et al. study of ES-SCLC patients with any response of RCT of PCI alone vs. PCI and thoracic RT showed a *non-significant difference in the primary end-point of 1 year OS (28% vs. 33%, p=0.066).* *On secondary analysis, a statistically significant 2-year OS improvement was noted (3% vs. 13%, p=0.004)*. Grade 3 or higher toxic effects occurred in 26 patients in the thoracic radiotherapy group and 18 patients in the control group (p=0·28) *All patients received PCI*, so this was not tested, but *historical data* suggests a survival benefit of *~5% for PCI,* less than the observed 10% benefit to chest RT in this study.

What is the recommended treatment in limited-stage-SCLC based on the phase III randomized study (CONVERT) of concurrent chemoRT with once-daily RT to 66 Gy vs. twice-daily RT to 45 Gy? a. Twice-daily RT, due to improved efficacy over once-daily RT b. Once-daily RT, due to improved efficacy over twice-daily RT c. Once-daily RT, due to improved toxicity over twice-daily RT d. Twice-daily RT, due to lack of superiority of once-daily RT

D The CONVERT trial was *designed to examine superiority of once-daily vs. twice daily RT*, and at a median follow-up of 45 months, median overall survival was 30 months (95% CI 24-34) in the twice-daily group versus 25 months (21-31) in the once-daily group (HR in the once daily group 1·18 [95% CI 0·95-1·45]; p=0·14). *Survival and toxicity did not differ*, but since the trial was *designed to show superiority* of once-daily RT and not powered for equivalence, authors *recommend twice-daily RT to be considered standard of care*. 1. CONVERT - Faivre-Finn (2017)- Concurrent once-daily versus twice-daily chemo-RT in patients with LS-SCLC -45Gy in 30fx (1.5 Gy BID) vs 66Gy in 33fx (2Gy daily) -Patients: LS-SCLC with good performance status (0-2) and adequate pulm fx. Median 45mo f/u -Results: OS 30mo vs 25mo, NS (BID vs QD); 2yr OS 56% vs 51%, NS -Toxicity: similar between 2 groups. Increases grade 4 neutropenia in BID (49% vs 38%), comparable esophagitis, pneumonitis -Conclusion: survival outcomes did not differ but powered for superiority study and not equivalence. If anything BID was slightly improved over daily. BID RT should remain standard of care.

Regarding the European phase III study evaluating the benefit of thoracic RT in patients with extensive stage SCLC (Slotman et al. 2015): (A) OS at 1 yr was significantly improved with the addition of thoracic RT. (B) OS at 2 yr was NOT improved with the addition of thoracic RT. (C) Only CR to chemotherapy were eligible. (D) The primary endpoint was NOT met.

D The Slotman, et al. study examined 498 patients with extensive stage SCLC. Patients with *any response to initial chemotherapy (4-6 cycles of platinum/etoposide)* were eligible. PCI was mandated for all patients. The thoracic radiation dose was 30Gy/10 fractions. *The primary endpoint, of 1 year overall survival was not met.* Median overall survival was also similar between the two arms. *Two year overall survival, however favored the addition of thoracic irradiation 13% vs 3%.* Rates of *thoracic progression* were significantly *lower in the thoracic radiation group (44% vs. 80%)*. 2015PMID: 25230595 "Use of thoracic radiotherapy for extensive stage small-cell lung cancer: a phase 3 randomized controlled trial" (Slotman BJ et al. Lancet 2015 Jan 3;385(9962):36-42) Outcome: Median FU 24 months. 1 year OS thoracic RT 33% vs 28% no thoracic RT (p=0.066). 2 year OS13% vs 3%(p=0·004). 6 month PFS 24% vs 7%(p=0·001) 2017PMID: 28625628 - Secondary analysis, "Which patients with ES-SCLC are most likely to benefit from more aggressive radiotherapy". Worse prognosis = liver, bone, or >2 mets. Best if <=2 met sites. Conclusion:"Thoracic radiotherapy in addition to prophylactic cranial irradiation should be considered for all patients with ES-SCLC who respond to chemotherapy."

How would the AJCC 7th and 8th editions of TNM staging system characterize T4 malignant pleural mesothelioma? a. Involvement of the endothoracic fascia b. Focal extension into the soft tissues of the chest wall c. Non-transmural involvement of the pericardium d. Direct extension of tumor to the contralateral pleura

D The definitions of T3 and T4 tumors *did not change between the 7th and 8th editions*. *T3* tumors are those that are *locally advanced but technically resectable*. *T4* tumors are *technically unresectable*. AJCC 8th edition T1-Tumor limited to the *ipsilateral parietal pleura* with or *without involvemen*t of • visceral pleura • mediastinal pleura • diaphragmatic pleura T2-Tumor involving each of the *ipsilateral pleural surfaces* (parietal, mediastinal, diaphragmatic, and visceral pleura) with at *least one of the following features*: • involvement of *diaphragmatic muscle* • extension of tumor from visceral pleura into the *underlying pulmonary parenchyma* T3-Describes locally advanced but *potentially resectable tumor.* Tumor involving *all the ipsilateral pleural surfaces* (parietal, mediastinal, diaphragmatic, and visceral pleura) with at *least one of the following features*: • involvement of the *endothoracic fascia* • extension into the *mediastinal fat* • solitary, *completely resectable* focus of tumor extending into the *soft tissues of the chest wall* • *nontransmural involvement of the pericardium* T4-Describes locally advanced *technically unresectable tumor.* Tumor involving *all the ipsilateral pleural surfaces* (parietal, mediastinal, diaphragmatic, and visceral pleura) with at least one of the following features: • *diffuse extension* or multifocal masses of tumor in the *chest wall*, with or without associated rib destruction • direct *transdiaphragmatic extension* of tumor to the peritoneum • direct extension of tumor to the *contralateral pleura* • direct extension of tumor to *mediastinal organs* • direct extension of tumor *into the spine* • tumor extending through to the *internal surface of the pericardium* with or without a pericardial effusion; or tumor involving the myocardium N1-Metastases in the *ipsilateral* bronchopulmonary, hilar, or mediastinal (including the internal mammary, peridiaphragmatic, pericardial fat pad, or intercostal) lymph nodes N2-Metastases in the *contralateral mediastinal*, ipsilateral, or contralateral *supraclavicular lymph nodes.*

What is the TN stage for a patient with a 5.1 cm right upper lobe adenocarcinoma of the lung, and biopsy-proven levels 4R, 5, and 7 lymph nodes? (A) T2bN2 (B) T2bN3 (C) T3N2 (D) T3N3

D This patient presents with a T3 (5-7 cm) tumor with bilateral mediastinal lymph nodes (N3). *Station 5 &6 for a right sided tumor is considered a contralateral node, therefore N3*. T1-Tumor ≤3 cm in greatest dimension, surrounded by lung or visceral pleura, without bronchoscopic evidence of invasion more proximal than the lobar bronchus (i.e., not in the main bronchus) T2-Tumor >3 cm but ≤5 cm or having any of the following features: • Involves the main bronchus regardless of distance to the carina, but without involvement of the carina • Invades visceral pleura (PL1 or PL2) • Associated with atelectasis or obstructive pneumonitis that extends to the hilar region, involving part or all of the lung T2 tumors with these features are classified as T2a if ≤4 cm or if the size cannot be determined and T2b if >4 cm but ≤5 cm. T3-Tumor >5 cm but ≤7 cm in greatest dimension or directly invading any of the following: parietal pleura (PL3), chest wall (including superior sulcus tumors), phrenic nerve, parietal pericardium; or separate tumor nodule(s) in the same lobe as the primary T4-Tumor >7 cm or tumor of any size invading one or more of the following: diaphragm, mediastinum, heart, great vessels, trachea, recurrent laryngeal nerve, esophagus, vertebral body, or carina; separate tumor nodule(s) in an ipsilateral lobe different from that of the primary N1-Metastasis in ipsilateral peribronchial and/or ipsilateral hilar lymph nodes and intrapulmonary nodes, including involvement by direct extension N2-Metastasis in ipsilateral mediastinal and/or subcarinal lymph node(s) N3-Metastasis in contralateral mediastinal, contralateral hilar, ipsilateral or contralateral scalene, or supraclavicular lymph node(s)

The tumor PD-L1 expression enrollment requirement in the KEYNOTE-024 Phase-III randomized study for untreated advanced NSCLC (pembrolizumab vs chemotherapy) was: a. No requirement b. 1%. c. 5%. d. 50%.

D To be eligible for the *Keynote-010 study*, patients with previously treated advanced NSCLC were required to have at *least 1% tumor cells with PD-L1 expression*. In the total patient population, *median OS in the two pembrolizumab dose arms was significantly longer than docetaxel*. In a *planned subset analysis* of patients with expression of *PD-L1 on 50%* of the tumor cells, *median OS and PFS* in the two pembrolizumab dose arms was significantly longer than docetaxel. In *KEYNOTE-024 phase 3 trial*, of untreated advanced NSCLC with *PD-L1 expression on at least 50%* of tumor cells and *no EGFR mutation or ALK translocation* treated with *pembrolizumab (200 mg every 3 weeks)* or choice of *platinum-based chemotherapy*.*Crossover from the chemotherapy group to the pembrolizumab group was permitted in *disease progression*. RESULTS Median *PFS was 10.3 months* in the pembrolizumab group *versus 6.0 months* in the chemotherapy group *HR 0.50*; increased response rate (44.8% vs. 27.8%) and the median duration of response was longer (not reached [range, 1.9+ to 14.5+ months] vs. 6.3 months [range, 2.1+ to 12.6+]), *treatment-related adverse events* of any grade were less frequent* (occurring in 73.4% vs. 90.0% of patients)*, as were *grade 3, 4, or 5 treatment-related adverse events (26.6% vs. 53.3%).

For a female patient with metastatic squamous cell carcinoma of the lung and a minimal smoking history, to which novel therapeutic is she MOST likely to respond? (A) Crizotinib (B) Ipilimumab (C) Erlotinib (D) Nivolumab

D lpilimumab has not been shown to be effective in lung cancer. *Erlotinib, Gefitinib, Afatinib and Crizotinib* have all been shown to be effective *in select Adenocarcinomas* who clinically have the typical presentation of a young Asian female non- smoker and of course now has an *EGFR mutation* or in *crizotnib's case has the gene rearrangement of ALK.* Nivolumab is the first novel agent approved for squamous cell carcinoma which is a fully human lgG4 programmed death 1(PD-1) immune- checkpoint- inhibitor antibody which doubled the response rate and doubled survival in comparison to docetaxel which is FDA approved in the second line setting.

Regarding early stage NSCLC: (A) the local control after lobectomy is better than SBRT. (B) there is a survival advantage of lobectomy over wedge resection. (C) there is a survival advantage seen for adjuvant chemotherapy in stage I. (D) there is no benefit of brachytherapy added to sublobar resection.

D. ACOSOG Z4032: 224 High- risk operable patients with NSCLC ≤ 3 cm randomized to sublobar resection (SR) or SR with brachytherapy. High risk based on ≥1 major criteria (FEV1 £ 50% predicted, DLCO £ 50% predicted) or ≥ 2 minor criteria (Age ³ 75, FEV1, DLCO, pulm HTN, poor LVEF, etc) - *Brachytherapy did not reduce LR (LP+lobe+N1 nodes) after SR. 3-year OS similar.* - May be due to closer attention to parenchymal margins by surgeons in this study (local progression (at staple line) 7.7%) A is not correct since local control is nearly equivalent between lobectomy (95%) and SBRT (98%-RTOG 0236); B is not correct since LACE shows only survival advantage for stage II-III resected NSCLC; C is not correct because LCSG921 only demonstrated better local control in lobectomy but not survival;

What was the key finding of INT 0160, a phase II trial exploring neoadjuvant chemoradiation followed by surgical resection for patients with superior sulcus NSCLC? (A) Local failure was the most common site of disease progression. (B) Most patients were unable to complete neoadjuvant chemoradiation. (C) Patients with T3 tumors had double the OS of those with T4 tumors. (D) Five-year OS for patients with a complete resection was 54%.

D. INT 0160/SWOG 9416: Induction chemoRT and surgical resection. very select group T3-4N0-1 NSCLC who received 2 cycles of cisplatin/etoposide concurrently with RT → if stable/response → 2 more cycles of chemotherapy. -pCR or minimal microscopic disease in 56%, -5-year OS for the entire group was 44% and 54% for those who underwent a complete resection*, which is nearly *double* that reported for those with N2 disease treated with *definitive chemoradiotherapy or neoadjuvant chemoradiotherapy. *Superior Sulcus Tumors Management*: -If operable, concurrent chemo-RT (45Gy)→surgery→chemo (preferred) -If marginally resectable, concurrent chemo-RT (45Gy)→restage→if no progression →surgery →chemo (INT0160) -If unresectable (initially or after restaging), complete definitive chemo-RT (63-66Gy) → durvalumab (PD-L1)

What was a finding of INT 0139, a phase III trial comparing neoadjuvant chemoradiation followed by surgical resection to definitive chemoradiation for patients with locally advanced NSCLC? (A) Both study arms received carboplatin and paclitaxel. (B) Most treatment related deaths occurred in lobectomy patients. (C) The majority of treatment-related deaths occurred during chemoradiation in both study arms. (D) Lobectomy after neoadjuvant chemoradiation had improved OS compared with definitive chemoradiation.

D. In this pivotal trial, only patients with stage IIIA (N2) NSCLC were eligible. *Progression-free survival was superior for neoadjuvant chemoradiotherapy* (median 12.8 vs. 10.5 mo, p=0.017), though *overall survival was not*. In an unplanned analysis, overall survival was improved for neoadjuvant chemoradiotherapy for patients who underwent LOBECTOMY but not pneumonectomy. *High rates of treatment related deaths occurred in patients undergoing PNEUMONECTOMY.* INT-0139- Radiotherapy plus chemotherapy with or without surgical resection for Stage III NSCLC- (Lancet 2009; ASCO 2005 abstr.) ● 396 patients with T1-3 pN2M0 (biopsy proven N2) ● Treated with concurrent chemo-RT x 2c +45Gy→restaging→if no progression randomized to surgery followed by chemo x 2C vs concurrent chemo-RT to 61Gy without surgery followed by chemo x 2C ● Chemo was cisplatinum and etoposide ● *18% of patients in surgery arm had pCR to induction chemo* ● Surgery *improved 5-year PFS (11→22%)* and median PFS (10.5→12.8 months) with fewer local only relapses (10 vs 22%) ● There was *no significant difference in MS* (23.6 vs 22.2 months, p=0.24), although there was a 5-year OS trend in favor of surgery (20 vs 27%). ● Increased treatment related deaths with surgery (8 vs 2%), particularly when pneumonectomy required ● Unplanned subset analysis: *lobectomy superior to chemo RT* (which included patients who would have undergone a pneumonectomy since we can't figure out who would have or have not had received pneumonectomy). OS improved in the surgical group if a lobectomy was done compared with rate in a matched chemorads group

Which change in the four R's of radiotherapy has been implicated in the success of SBRT for early-stage NSCLC? (A) Limited tumor reoxygenation (B) Inhibited tumor stem cell repopulation (C) Disrupted cell-cycle redistribution (D) Incomplete sublethal damage recovery

D. Most tumors are characterized by α/β ratios of at least 10 Gy, which makes them less sensitive to changes in dose fractionation than most late responding normal tissues (α/β ratios of approximately 3 Gy). Nevertheless, even tumors with high α/β ratios will still be affected by the large fraction sizes associated with SBRT due to incomplete repair of sublethal damage. The short overall treatment times with SBRT could also be advantageous in that subset of tumors that have the ability to undergo accelerated repopulation over a several- week course of fractionated radiation therapy. However, reoxygenation of hypoxic tumors could be limited during a short course of SBRT, which is a disadvantage of the technique and could lead to local relapse. The question of whether the large fraction sizes used in SBRT invoke a "new (radio)biology" is a matter of active research and debate.

Lung cancer screening should be recommended for which of the following individuals? (A) 45-year-old current smoker with 20-pack-year history of smoking (B) 50-year-old non-smoker with strong family history of lung cancer (C) 75-year-old with 35-pack-year history of smoking, but quit 20 years ago (D) 65-year-old with 30-pack-year history of smoking, but quit 10 years ago

D. The risk factors are age *55 to 74 years, 30 or more pack-year history* of smoking tobacco, and if *former smoker, have quit within 15 years.*

The risk of NSCLC in a smoker working in the asbestos industry increases by what factor? (A) 5 X (B) 10 X (C) 30 X (D) 60 X

D. The risk of developing mesothelioma was described as *10% over the lifetime of an asbestos worker*, with up to *70% of all mesothelioma cases involving documented asbestos exposure.* Concomitant smoking enhances the risk of malignancy in an asbestos worker, with a *60-fold increased* risk of developing non-small cell lung cancer. The *chance of dying of a malignancy* (mesothelioma or lung cancer) versus a nonmalignant cause is *50% in an individual exposed to asbestos* compared to *18% in an individual not exposed*. Asbestos workers are at highest risk, but family members can also be at risk via exposure to fibers brought home on the clothing of the primary individual.

According to the updated QUANTEC, what mean dose (Gy) to the whole lung with 3D-CRT would result in a 5% risk of symptomatic pneumonitis? (A) 7 (B) 13 (C) 20 (D) 24

A. The risk of symptomatic radiation pneumonitis increases with the dose of radiation. *Mean dose to whole lung*: 7 Gy-5% risk, 13 Gy-10%, 20 Gy-20% 24 Gy 30%. To limit V20 < 30% would have the risk less than 20% risk

The spinal cord maximum dose constraint when planning definitive twice-daily radiotherapy for patients with limited-stage SCLC with modern radiation techniques is: a. 36 Gy. b. 41 Gy. c. 45 Gy. d. 48 Gy.

B Based on the CALGB 30610/RTOG 0538 protocol, the max dose constraint on the spinal cord for patients receiving definitive chemoradiation to *45 Gy in 30 fractions BID should not exceed 41 Gy*.

Based on the results of RTOG 0617 (Bradley 2015), which of the following cardiac dose constraints were found to be predictive of OS on multivariate analysis? (A) V20 (B) V30 (C) V45 (D) V60

B The RTOG 0617 was a double randomized study looking at 60 vs 74Gy as well as cisplatin +/- cetuximab for the treatment of locally advanced Stage IIIA-B NSCLC. the 74Gy arm had worse survival. One of the important findings of the study was that *cardiac V5 and V30 were both predictive of mortality on multivariate analysis.* This has been incorporated into RTOG 1308 where the *cardiac constraints were now lowered to V30<50%*.

What is the maximum dose constraint in Gy for the spinal cord when SBRT is given in 3 fractions for an early stage lung cancer? (A) 14 (B) 18 (C) 26 (D) 30

B. Based on constraints used in recent and ongoing RTOG SBRT trials (0618, 0813, and 0915). -Less than 14 Gy for single fraction, *-18 Gy for 3 fractions* -26 Gy for 4 fractions and -30 Gy for 5 fractions of SBRT

A palliative, AP/PA radiation plan is created for a patient with large mediastinal mass and 30 Gy in 10 fractions. What acute complication is most likely if the fields incorporate the mediastinum, the whole heart, the lower trachea, carina and mainstem bronchi, 15% of total lung volume, and the lower 1/3 of the esophagus? (A) Bronchial stenosis (B) Radiation fibrosis (C) Radiation pericarditis (D) Esophageal stricture

C *Bronchial stenosis* is a complication more commonly seen with *SBRT courses to central lesions* and is a late complication. *Radiation pneumonitis is unlikely as the V20 would only be 15%.* Per QUANTEC reports, the symptomatic pneumonitis rate when *V20 is <30% is<20%.* *Esophageal stricture* is a late complication, unlikely to be seen at this low of a dose. *Radiation to the whole heart* is associated with *high rate of radiation pericarditis, up to 46%* in Hodgkin's Lymphoma data before the LV and subcarinal blocks were introduced in mantle fields.

What was the radiation dose on the experimental arm of the CONVERT trial presented at the 2016 ASCO that used 45 Gy (1.5 Gy twice daily) as the standard arm for limited stage SCLC? (A) 45 Gy (B) 60 Gy (C) 66 Gy (D) 70 Gy

C *CONVERT trial* is an international randomized trial of concurrent chemo-radiotherapy *comparing twice daily (45 Gy) and daily (66 Gy) radiotherapy* schedules in patients with limited-stage SCLC and good performance status. Results were presented at the 2016 ASCO meeting showing a *2-yr OS of 56% vs 51% and median OS* of 30m vs 25 m (p=0.15) with *no differences in toxicity* supporting the use of either regiment for standard of care treatment of LS-SCLC with good PS. *was not a non-inferiority trial* so the BID treatment from the Turrissi trail remains the standard of care.* The standard-arm in Turrissi, et al. Trial was *45 Gy daily* *CALGB 30610* is an ongoing trial comparing *45 Gy BID vs 70 Gy OD* (61.2 Gy concomitant boost arm was closed early).

A nearly spherical lung lesion treated with EBRT has a prescription isodose volume (PIV) to target volume ratio (PITV) of 1.3. If the PIV is 148 cc, what is the estimated diameter of the target? (A) 2cm (B) 4cm (C) 6cm (D) 8cm

C If the PIV is 148cc, the target volume is 114cc. From that volume (4/3 π r^3) you can determine the radius (and then diameter) of the sphere. The estimated diameter for a sphere is D = ((3xπ) / (32xV))^(1/3).

Regarding RTOG 0617 for lung cancer, what were the per protocol constraints for total lungs minus CTV? (A) V20 ≤ 25% and mean dose ≤ 15 Gy (B) V20 ≤ 25% and mean dose ≤ 20 Gy (C) V20 ≤ 37% or mean dose ≤ 15 Gy (D) V20 ≤ 37% or mean dose ≤ 20 Gy

D The per protocol lung constraint in RTOG 0617 was for total (bilateral) lungs minus CTV. The recommended constraints were *V20 ≤ 37% or alternatively a mean dose ≤ 20 Gy*.

Which of the following statements was TRUE in the combined analysis of the surgery vs. SBRT trials for early stage NSCLC (STARS and ROSEL; Chang, et al. Lancet Oncology 2015)? (A) LC was higher in the surgery arm (B) OS was higher in the surgery arm (C) LC was higher in the SBRT arm (D) OS was higher in the SBRT arm

D This study combined patients from two incompletely accruing SBRT vs. surgery trials for *early stage NSCLC (cT1-T2a < 4 cm) in medically operable patients.* It included only 58 patients, however contains much important data. There was *no difference in local control, regional control, or distant control between surgery and SBRT*. There were *no grade 4-5 toxicities in the SBRT arm*. One patient died of surgical complications in the surgery arm. There was no difference in recurrence free survival (86% in the SBRT group vs. 80% in the surgery group, p = ns). There was, however, a difference in 3 year overall survival (95% in the SBRT group vs. 79% in the surgery group).


संबंधित स्टडी सेट्स

Biology chemical reactions chapter 6 section 2&4 k

View Set

Section 5: Statutes, rules common to life and health

View Set

NUR 195 Nursing Assessment: Hematologic Function

View Set

Emergency Care 13th Addition Section 3

View Set

plastic research paper note cards

View Set